Tag Archives: Lop9

PHƯƠNG TRÌNH NGHIỆM NGUYÊN DẠNG LUỸ THỪA

A. MỘT SỐ CHÚ Ý KHI GIẢI PHƯƠNG TRÌNH DẠNG LŨY THỪA
Nhận xét: Để giải phương trình nghiệm nguyên dạng lũy thừa ta chú ý một số phương pháp thường sử dụng

  • Sử dụng đồng dư để xét tính chẵn lẻ, hay modun của nghiệm.
  • Phân tích thành thừa số.
  • Đánh giá bất đẳng thức.

Do sử dụng nhiều đồng dư, do đó ta chú ý một số tính chất về đồng dư sau Tính chất 3.2. Cho $a$ là một số nguyên tùy ý. Khi đó
(a) $a^2 \equiv 0,1(b\mod 3)$;
(b) $a^2 \equiv 0,1(b\mod 4)$
(c) $a^2 \equiv 0,1,4 (b\mod 8)$;
(d) $a^2 \equiv 0,1,4 (b\mod 5)$;
(e) $a^3 \equiv-1,0,1 (b\mod 7)$
(f) $a^3 \equiv-1,0,1(b\mod 9)$.

Tính chất 3.3. Cho $p$ là một số nguyên tố và $a, b, c, n$ là các số nguyên dương. Ta có
(a) $a^n \vdots p \Leftrightarrow a \vdots p$;
(b) Nếu $a b=p^n$ thì $\left\{\begin{array}{l}a=p^k \\\ b=p^{n-k}\end{array} \quad\right.$ với $k \in \mathbb{N}$ thỏa $0 \leq k \leq n$;
(c) Nếu a b=c^n và (a, b)=1 thì $a=s^n \text { và } b=r^n$ với $s, r \in \mathbb{N}$.

B MỘT SỐ VÍ DỤ
Ví dụ 3.29. Tìm các số nguyên $x, y$ thỏa mān $x^3+1=4 y^2$.

Hướng dẫn giải

Giả sử tồn tại các số nguyên $x, y$ thỏa mãn $x^3+1=4 y^2$. Ta có
$$
x^3=4 y^2-1=(2 y-1)(2 y+1) \text {. }
$$

Đặt $d=(2 y-1,2 y+1)$, ta có $d$ lẻ và $\left\{\begin{array}{l}d \mid 2 y-1 \\\ d \mid 2 y+1\end{array}\right.$.
Do đó $d \mid 2$, suy ra $d=1$ (vì $d$ lẻ). Như vậy $2 y-1$ và $2 y+1$ nguyên tố cùng nhau.
Kết hợp với (3.1) ta suy ra $2 y-1=a^3$ và $2 y+1=b^3$ với $a, b \in \mathbb{Z}$.
Dẫn đến $b^3-a^3=2$ hay $(b-a)\left(b^2+b a+a^2\right)=2$. Từ đó ta được $b=1$ và $a=-1$, suy ra $y=0$ và khi đó $x=-1$. Thử lại thỏa.
Vậy $(x, y)=(-1,0)$.

Ví dụ 3.30. Giải phương trình nghiệm nguyên $x^5+2023 x=5^y+2$.

Hướng dẫn giải

Giả sử tồn tại các số nguyên $x, y$ thỏa mãn $x^5+2023 x=5^y+2$.
Vì $5^y+2$ lẻ nên $x$ lẻ, do đó $x^5+2023 x=x\left(x^4+2023\right) \vdots 4$ (vì $x$ lẻ nên $x \equiv 1(\bmod 4)$.
Tuy nhiên $x^5+2023 x=5^y+2 \equiv 1^y+2 \equiv 3(\bmod 4)$ (Vô lí).

Vậy không tồn tại các số nguyên $x, y$ thỏa mãn $x^5+2023 x=5^y+2$.

Ví dụ 3.31. Tìm các số nguyên $x$ và $y$ sao cho $3^x-y^3=1$.

Hướng dẫn giải

Giả sử tồn tại các số nguyên $x$ và $y$ sao cho $3^x-y^3=1$. Nhận xét $x \geq 0$.
Ta có $3^x=y^3-1=(y+1)\left(y^2-y+1\right)$, suy ra $\left\{\begin{array}{l}y+1=3^t \\\ y^2-y+1=3^{x-t}\end{array} \quad(t \in \mathbb{N}, t \leq x)\right.$.
Khi đó $y=3^t-1$ và
$$
\left(3^t-1\right)^2-\left(3^t-1\right)+1=3^{x-t} \Leftrightarrow 3^{2 t}-3^{t+1}+3=3^{x-t} .
$$

  • Nếu $t=0$, từ (3.2) ta được $1=3^x$ hay $x=0$. Ngoài ra $y=3^0-1=2$.

Nếu $t \geq 1$, giả sử $x-t \geq 2$, khi đó $3^{x-t} \vdots 9$. Từ (3.2) ta có $3^{2 t} \vdots 9$ và $3^{t+1} \vdots 9$ (do $t \geq 1$ ), từ đó suy ra $3 \vdots 9$ (Vô lí).
Do đó $x-t \in{0,1}$.

  • Nếu $x-t=0$ thì $y^2-y+1=1 \Leftrightarrow y(y-1)=0 \Leftrightarrow\left[\begin{array}{l}y=0 \ y=1\end{array}\right.$.
    Với $y=0$ ta tìm được $x=0$ và với $y=1$ ta có $3^x=2$ (Vô lí).
  • Nếu $x-t=1$ thì $y^2-y+1=3 \Leftrightarrow y^2-y-2=0 \Rightarrow y=2$.
    Khi đó $3^x=2^3+1=9$, dẫn đến $x=2$.

Vậy $(x, y)=(0,0)$ hoặc $(x, y)=(2,1)$.

Ví dụ 3.32. Tìm các số nguyên dương $x$ và $y$ sao cho
$$
9^x-7^x=2^y .
$$

Hướng dẫn giải

Giả sử tồn tại các số nguyên dương $x, y$ sao cho $9^x-7^x=2^y$.
Nếu $x$ lẻ thì
$$
9^x-7^x \equiv 1^x-(-1)^x \equiv 2(\bmod 8) .
$$

Do đó $2^y \equiv 2(\bmod 8)$, suy ra $y=1$. Khi đó $9^x-7^x=2 \Rightarrow x=1$.
Nếu $x$ chẵn, đặt $x=2 k\left(k \in \mathbb{N}^*\right)$, ta được
$$
2^y=9^{2 k}-7^{2 k}=\left(9^k-7^k\right)\left(9^k+7^k\right) .
$$

Suy ra
$$
\left\{\begin{array}{l}
9^k-7^k=2^t \\
9^k+7^k=2^{y-t}
\end{array}\right.
$$
với $t \in \mathbb{N}^*$ và $t \leq y$.
– Nếu $k$ lẻ, khi đó $2^t \equiv 9^k-7^k \equiv 2(\bmod 8)$, do đó $t=2$ và $k=1$.
Dẫn đến $x=2$ và $2^y=81-49=32 \Rightarrow y=5$.
– Nếu $k$ chẵn, ta có
$$
9^k+7^k \equiv 1^k+(-1)^k \equiv 2(\bmod 8) .
$$

Do đó $2^{y-t} \equiv 2(\bmod 8)$, suy ra $y-t=1$. Như vậy $9^k+7^k=2$ (Vồ lí).
Vậy $(x, y)=(1,1)$ hoặc $(x, y)=(2,5)$.

Ví dụ 3.33. Tìm tất cả các số nguyên tố $p$ sao cho luôn tồn tại các số nguyên dương $n, x, y$ thỏa mãn
$$
p^n=x^3+y^3 .
$$

Hướng dẫn giải

Đặt $x=p^t x_1$ và $y=p^s y_1\left(x_1, y_1, s, t \in \mathbb{N}\right.$ và $\left.x_1, y_1 \neq p\right)$.
Ta có
$$
p^n=p^{3 t} x_1^3+p^{3 s} y_1^3>p^{3 t} \Rightarrow n>3 t .
$$

Không mất tính tổng quát, giả sử $t \geq s$.
Nếu $t>s$ thì $p^{n-3 s}=p^{3(t-s)} x_1^3+y_1^3 \vdots p \Rightarrow y_1^3 \vdots p$ (Vô lí).
Vậy $t=s$, do đó $p^{n-3 t}=x_1^3+y_1^3=\left(x_1+y_1\right)\left(x_1^2-x_1 y_1+y_1^2\right)$.

  • Nếu $x_1^2-x_1 y_1+y_1^2=1$ thì $x_1=y_1=1$.
    Khi đó $p^{n-3 t}=2 \Rightarrow\left\{\begin{array}{l}p=2 \\\ n-3 t=1\end{array} \Rightarrow\left\{\begin{array}{l}p=2 \\\ n=3 t+1\end{array}\right.\right.$.
    Lúc này ta được $x=y=2^t$. Thử lại thỏa.
  • Nếu $x_1^2-x_1 y_1+y_1^2>1$, ta được
    $$
    \left\{\begin{array}{l}
    x_1+y_1=p^k \\\
    x_1^2-x_1 y_1+y_1^2=p^{n-3 t-k}
    \end{array}\right.
    $$
    với $k \geq 1, n-3 t-k \geq 1$.

Do đó $\left(x_1+y_1\right)^2-\left(x_1^2-x_1 y_1+y_1^2\right)=3 x_1 y_1 \vdots p \Rightarrow 3 \vdots p \Rightarrow p=3$.

Ngoài ra, nếu $n-3 t-k \geq 2$ thì $x_1^2-x_1 y_1+y_1^2=\left(x_1+y_1\right)^2-3 x_1 y_1 \vdots 3^2$, mà $\left(x_1+y_1\right)^2 \vdots 3^2$ nên $3 x_1 y_1 \vdots 3^2 \Rightarrow x_1 y_1 \vdots 3$ (Vô lí).
Vậy $n-3 t-k=1$ hay $x_1^2-x_1 y_1+y_1^2=3$. Không mất tính tổng quát, giả sử $x_1 \geq y_1$ thì ta được $x_1=2$ và $y_1=1$.
Từ đây ta được $n-3 t=2 \Leftrightarrow n=3 t+2$ và $x=2 \cdot 3^t$ và $y=3^t$.
Thử lại thỏa.
Vậy $p=2$ và $p=3$ là các số nguyên tố cần tìm.

Ví dụ 3.34. Tìm nghiệm tự nhiên của phương trình
$$
\left(2^x+1\right)\left(2^x+2\right)\left(2^x+3\right)\left(2^x+4\right)-5^y=11879 .
$$

Hướng dẫn giải

Giả sử tồn tại các số tự nhiên $x, y$ thỏa mãn
$$
\left(2^x+1\right)\left(2^x+2\right)\left(2^x+3\right)\left(2^x+4\right)-5^y=11879 .
$$

Ta có
$$
\begin{aligned}
\left(2^x+1\right)\left(2^x+2\right)\left(2^x+3\right)\left(2^x+4\right) & =\left(4^x+5 \cdot 2^x+4\right)\left(4^x+5 \cdot 2^x+6\right) = \left(4^x+5 \cdot 2^x+5\right)^2-1 .
\end{aligned}
$$

Do đó $\left(4^x+5 \cdot 2^x+5\right)^2-1-5^y=11879 \Leftrightarrow\left(4^x+5 \cdot 2^x+5\right)^2-5^y=11880$.
Nếu $y \geq 1$ thì ta suy ra $4^x+5 \cdot 2^x+5 \vdots 5 \Rightarrow 4^x \vdots 5$. (Vô lí)
Do đó $y=0$, khi đó
$$
\left(4^x+5 \cdot 2^x+5\right)^2=11881 \Rightarrow 4^x+5 \cdot 2^x+5=109 \Leftrightarrow 4^x+5 \cdot 2^x-104=0 .
$$

Suy ra $2^x=8 \Rightarrow x=3$.
Vậy $x=3$ và $y=0$.

Ví dụ 3.35. Cho $M=a^2+3 a+1$ với $a$ là số nguyên dương.
(a) Chứng minh rằng mọi ước của $M$ đều là số lẻ.
(b) Tìm các giá trị của $a$ để $M$ là lũy thừa của 5 .

Hướng dẫn giải

(a) Ta có $a^2+3 a+1=a(a+3)+1$ là số lẻ. Do đó mọi ước của $M$ đều là số lẻ.
(b) Giả sử tồn tại $n \in \mathbb{N}^*$ thỏa mãn $a^2+3 a+1=5^n$. Khi đó
$$
a^2+3 a-4=5^n-5 \Leftrightarrow(a+4)(a-1)=5\left(5^{n-1}-1\right) .
$$

Nếu $n>1$ thì $5^{n-1}-1>0$.
Ta lại có $(a+4)(a-1) \vdots 5$ và $a+4-(a-1)=5$ nên $\left\{\begin{array}{l}a+4 \vdots 5 \\\ a-1 \vdots 5\end{array}\right.$.
Do đó $(a+4)(a-1) \vdots 25 \Rightarrow 5\left(5^{n-1}-1\right) \vdots 25 \Rightarrow 5^{n-1}-1 \vdots 5$. (Vô lí)
Vậy $n=1$ hay $a^2+3 a+1=5 \Rightarrow a=1$.
Thử lại thỏa, vậy $M$ là lũy thừa của 5 khi và chỉ khi $a=1$.

Ví dụ 3.36. Tìm số tự nhiên $n$ sao cho $8^n+47$ là số nguyên tố.

Hướng dẫn giải

  • Xét $n=2 k(k \in \mathbb{N})$, khi đó
    $$
    p^n \equiv 8^n+47 \equiv(-1)^{2 k}+47 \equiv 48 \equiv 0(\bmod 3) .
    $$

Do đó $p$ ! 3 nên $p$ không là số nguyên tố (Vô lí).

  • Xét $n=4 k+1\left(k \in \mathbb{N}^*\right)$, khi đó
    $$
    p \equiv\left(8^4\right)^k \cdot 8+47 \equiv 8+47 \equiv 55 \equiv 0(\bmod 5) .
    $$

Do đó $p \vdots: 5$ nên $p$ không là số nguyên tố (Vô lí).

  • Nếu $n=4 k+3\left(k \in \mathbb{N}^*\right)$, khi đó
    $$
    p \equiv\left(8^4\right)^k \cdot 8^3+47 \equiv 8^3+47 \equiv 559 \equiv 0(\bmod 13) .
    $$

Do đó $p$ : 13 nên $p$ không là số nguyên tố (Vô lí).
Vậy không tồn tại số tự nhiên $n$ để $8^n+47$ là số nguyên tố.

Ví dụ 3.37. Cho phương trình $2^x+5^y=k^2$ ( $x, y, k$ là các số nguyên dương).
(a) Chứng minh rằng phương trình trên vô nghiệm khi $y$ chẵn.
(b) Tìm $k$ để phương trình có nghiệm.
(Đề thi tuyển sinh vào lớp 10 chuyên toán PTNK 2022)

Hướng dẫn giải

(a) Giả sử tồn tại $y \in \mathbb{N}^*$ chẵn để phương trình trên có nghiệm.

  • Với $x=1$ thì $2+5^y=k^2 \equiv 2(\bmod 5)$.
    Điều này vô lý vì $k^2 \equiv 0,1,4(\bmod 5)$ với mọi $k \in \mathbb{N}$.
  • Với $x>1$, do $y$ chẵn nên ta đặt $y=2 m(m \in \mathbb{N})$.
    Khi đó
    $$
    2^x+5^{2 m}=k^2 \Leftrightarrow 2^x=\left(k-5^m\right)\left(k+5^m\right) \Rightarrow\left\{\begin{array}{l}
    k-5^m=2^t \\\
    k+5^m=2^{x-t}
    \end{array} \quad(t \geq 0) .\right.
    $$

Vì $k+5^m>k-5^m$ nên $x-t>t$, suy ra $k=2^{t-1}+2^{x-t-1}$.
Ta thấy nếu $t=0$ thì $k=\dfrac{1}{2}+2^{x-1} \notin \mathbb{N}$. Do đó $t \geq 1$.

Mặt khác $k$ lẻ và $t-1<x-t-1$ nên $2^{t-1}=1 \Rightarrow t=1$. Khi đó $k-5^m=2 \Leftrightarrow k=2+5^m$. Thay vào $2^x+5^{2 m}=k^2$, ta được
$$
2^x+5^{2 m}=\left(2+5^m\right)^2 \Leftrightarrow 2^x=4+2 \cdot 5^m .
$$

Vì $x>1$ nên $2^x \vdots 4$, suy ra $2 \cdot 5^m \vdots 4$ (Vô lí).
Vậy phương trình vô nghiệm khi $y$ chẵn.
(b) Giả sử phương trình có nghiệm, khi đó $y$ lẻ.

  • Nếu $x=4 z+1(z \in \mathbb{N})$ thì
    $$
    k^2 \equiv 2^x+5^y \equiv 2^{4 z} \cdot 2+5^y \equiv 2(\bmod 5) .
    $$

Điều này vô lý vì $k^2 \equiv 0,1,4(\bmod 5)$ với mọi $k \in \mathbb{N}$.

  • Nếu $x=4 z+3(z \in \mathbb{N})$ thì
    $$
    k^2 \equiv 2^{4 z} \cdot 2^3+5^y \equiv 8 \equiv 3(\bmod 5) \text { (Vô lí). }
    $$

Vậy $x$ chẵn, đặt $x=2 t\left(t \in \mathbb{N}^*\right)$.
Ta có
$$
2^x+5^y=k^2 \Leftrightarrow 5^y=\left(k-2^t\right)\left(k+2^t\right) \Rightarrow\left\{\begin{array}{l}
k-2^t=5^s \\\
k+2^t=5^{y-s}
\end{array} \quad(s \in \mathbb{N}) .\right.
$$

Nếu $s>0$ thì $5^{y-s}-5^s \vdots 5$ nên $2^{t+1} \vdots 5$ (vô lý). Do đó $s=0$.

Khi đó $\left\{\begin{array}{l}k=1+2^t \\\ k=5^y-2^t\end{array}\right.$. Suy ra $1+2^t=5^y-2^t \Rightarrow 5^y-1=2^{t+1}$.
Nếu $t>1$ thì $2^{t+1} \vdots 8$. Dặt $y=2 l+1$, khi đó
$$
2^{t+1}=5^y-1=25^l \cdot 5-1 \equiv 5-1 \equiv 4(\bmod 8) \text{vô lý}
$$

Vậy $t=1$, suy ra $k=3$. Với $k=3$, ta tìm được $x=2$ và $y=1$.
Vậy phương trình có nghiệm khi và chỉ khi $k=3$.

Ví dụ 3.38. Cho $k$ là số nguyên dương và $a=3 k^2+3 k+1$.
(a) Chứng minh rằng $2 a$ và $a^2$ là tổng của ba số chính phương.
(b) Chứng minh rằng nếu $a$ là uớc của số nguyên $b$ và $b$ bằng tổng của ba số chính phương thì bất kì lũy thừa với số mũ nguyên dương nào của $b$ cũng là tổng của ba số chính phương.

Hướng dẫn giải

(a) Ta có
$$
\begin{aligned}
2 a=6 k^2+6 k+2 & =k^2+\left(k^2+2 k+1\right)+\left(4 k^2+4 k+1\right) = k^2+(k+1)^2+(2 k+1)^2
\end{aligned}
$$
$$
\begin{aligned}
a^2 & =\left(3 k^2+3 k-1+2\right)^2=9 k^4+18 k^3+15 k^2+6 k+1 = \left(4 k^4+12 k^3+13 k^2+6 k+1\right)+\left(4 k^4+4 k^3+k^2\right)+\left(k^4+2 k^3+k^2\right) = \left(2 k^2+3 k+1\right)^2+\left(2 k^2+k\right)^2+\left(k^2+k\right)^2
\end{aligned}
$$
(b) Đặt $a^2=a_1^3+a_2^3+a_3^3$ với $a_1, a_2, a_3 \in \mathbb{Z}$.
Đặt $b=c a$ với $c$ là số nguyên dương, do $b$ bẳng tổng của ba số chính phương nên $b=b_1^2+b_2^2+b_3^2$ với $b_1, b_2, b_3$ là các số nguyên.
Xét số nguyên dương $n$ bất kì, khi đó

  • Nếu $n=2 k\left(k \in \mathbb{Z}^{+}\right)$thì
    $$
    \begin{aligned}
    b^n & =c^{2 k} a^{2 k}=\left(c^k a^{k-1}\right)^2 a^2 = \left(c^k a^{k-1}\right)^2\left(a_1^2+a_2^2+a_3^2\right) = \left(c^k a^{k-1} a_1\right)^2+\left(c^k a^{k-1} a_2\right)^2+\left(c^k a^{k-1} a_3\right)^2
    \end{aligned}
    $$
  • Nếu $n=2 k+1(k \in \mathbb{Z})$ thì
    $$
    b^n=\left(b^k\right)^2 \cdot b=\left(b^k\right)^2\left(b_1^2+b_2^2+b_3^2\right)=\left(b^k b_1\right)^2+\left(b^k b_2\right)^2+\left(b^k b_3\right)^2
    $$

Hoàn tất chứng minh.


C. CÁC BÀI TẬP RÈN LUYỆN

Bài 3.13. Tìm nghiệm nguyên dương của phương trình
$$
x^3+x^2+x+1=2011^y .
$$

Bài 3.14. Tìm tập nghiệm nguyên dương của phương trình
$$
8^x+15^y=17^z .
$$

Bài 3.15. Tìm các số nguyên dương $x, y, z>1$ thỏa mãn
$$
(x+1)^y-x^z=1 .
$$

Bài 3.16. Tìm nghiệm tự nhiên của phương trình $5^x-3^y=2$.

Bài 3.17. Tìm nghiệm nguyên dương của phương trình
$$
2^x \cdot 3^y+5^z=7^t .
$$

Bài 3.18. Cho các số nguyên dương $m, n \geq 2$. Tìm nghiệm nguyên dương của phương trình
$$
x^n+y^n=3^m .
$$

Bài 3.19. Cho $p$ là một số nguyên tố và $a, n$ là các số nguyên dương. Chứng minh rằng nếu $2^p+3^p=$ $a^n$ thì $n=1$.

Bài 3.20. Chứng minh rằng tích của ba số nguyên liên tiếp không thể là lũy thừa với số mũ lớn hơn 1 của một số nguyên.

Bài 3.21. Cho phương trình $3 x^2-y^2=23^n$ với $n$ là số tự nhiên.
(a) Chứng minh nếu $n$ chẵn thì phương trình đã cho không có nghiệm nguyên $(x, y)$.
(b) Chứng minh nếu $n$ lẻ thì phương trình đã cho có nghiệm nguyên $(x, y)$.

Bài 3.22.
(a) Cho $m$ là số nguyên. Chứng minh rằng nếu tồn tại các số nguyên $a, b, c$ khác 0 sao cho $a+b+c=0$ và $a b+b c+c a+4 m=0$ thì cũng tồn tại các số nguyên $a^{\prime}, b^{\prime}, c^{\prime}$ sao cho $a^{\prime}+b^{\prime}+c^{\prime}=0$ và $a^{\prime} b^{\prime}+b^{\prime} c^{\prime}+a^{\prime} c^{\prime}+m=0$.
(b) Với $k$ là số nguyên dương, chứng minh rằng không tồn tại các số nguyên $a, b, c$ khác 0 sao cho $a+b+c=0$ và $a b+b c+c a+2^k=0$.
(Đề thi tuyển sinh lớp 10 chuyên Toán PTNK 2015)

ƯỚC CHUNG VÀ MỘT SỐ ÁP DỤNG

Bài viết của thầy Nguyễn Vĩnh Khang – Giáo viên Star Education

Các tính chất của ước chung

Nhận xét: Nếu ta đặt $(x, y)=d$, thì $x^{\prime}=\dfrac{x}{d}$ và $y^{\prime}=\dfrac{y}{d}$ nguyên tố cùng nhau. Từ đó lợi dụng các tính chất liên quan đến số nguyên tố cùng nhau như (được sử dụng thẳng, không cần chứng minh)

  • Nếu $a b: c$, và $(b, c)=1$, ta có $a: c$.
  • Nếu $a: b$ và $a: c$, với $(b, c)=1$, ta có $a: b c$.
  • Nếu $(a, b)=1, r$ là ước của $a, s$ là ước của $b$, ta cũng có $(r, s)=1$.
    để phân tích bài toán tiếp. Việc đặt ước chung như vậy sẽ làm đơn giản bài toán (do ta có thể rút $d$ ra rồi triệt tiêu, nếu được) và cho thêm dữ kiện $\left(x^{\prime}, y^{\prime}\right)=1$.

Tính chất 3.1. Giả sử $a, b, c, n$ là các số nguyên dương, chứng minh những tính chất sau
(a) $\operatorname{gcd}(a, b, c)=\operatorname{gcd}(\operatorname{gcd}(a, b), c)$
(b) $\operatorname{gcd}(a c, b c)=\operatorname{gcd}(a, b) c$
(c) Nếu $\operatorname{gcd}(a, b)=1$, ta có $\operatorname{gcd}(a b, c)=\operatorname{gcd}(a, c) \operatorname{gcd}(b, c)$
(d) $\operatorname{gcd}\left(a^n, b^n\right)=\operatorname{gcd}(a, b)^n$.

Chứng minh.
Phần 1: gọi $d=\operatorname{gcd}(a, b, c)$ ta có $d$ là ước của $a, b$, nên $\operatorname{gcd}(a, b)$ : $d$. Nhưng $c: d$, nên ta được một chiều
$$
\operatorname{gcd}(\operatorname{gcd}(a, b), c) \vdots d=\operatorname{gcd}(a, b, c)
$$

Để chứng minh chiều còn lại, gọi $d=\operatorname{gcd}(\operatorname{gcd}(a, b), c)$. Tương tự như trên ta có $d$ là ước của $\operatorname{gcd}(a, b)$, nên $d$ cũng là ước của $a, b$. Nhưng $d$ là ước của $a, b, c$, nên
$$
\operatorname{gcd}(a, b, c) \vdots d=\operatorname{gcd}(\operatorname{gcd}(a, b), c)
$$

Kết hợp (1.1) và (1.2), ta có đpcm.

Phần 2: nếu $d=(a c, b c)$, ta có $d: c$ do $c$ là ước chung của $a c, b c$. Đặt $d=k c$, ta có $(a c, b c)=k c$, và $a c, b c: k c$. Nói cách khác $a, b: k$, nên $(a, b): k$, và
$$
c(a, b) \vdots k c=(a c, b c)
$$

Mặt khác, đặt $k=(a, b)$, ta có $a, b: k$, nên $a c, b c: k c$. Theo định nghīa, $(a c, b c) \vdots k c=(a, b) c$. Kết hợp với (2.1) ta có đpem $\operatorname{gcd}(a c, b c)=\operatorname{gcd}(a, b) c$.

Phần 3: gọi $k=\operatorname{gcd}(a, c), l=\operatorname{gcd}(b, c)$, theo tính chất 2 , ta được
$$
\left\{\begin{array}{l}
\operatorname{gcd}\left(\dfrac{a}{k}, \dfrac{c}{k}\right)=1 \\
\operatorname{gcd}\left(\dfrac{b}{l}, \dfrac{c}{l}\right)=1
\end{array}\right.
$$

Mặt khác $a: k, b: l$, nhưng $a, b$ lại nguyên tố cùng nhau, nên $k, l$ cūng vậy. Kết hợp với $c: k, l$, ta có $c: k, l$. Để ý rằng $\dfrac{c}{k l}$ là ước của $\dfrac{c}{k}$ và $\dfrac{c}{l}$, nên
$$
\left\{\begin{array}{l}
\operatorname{gcd}\left(\dfrac{a}{k}, \dfrac{c}{k l}\right)=1 \\
\operatorname{gcd}\left(\dfrac{b}{l}, \dfrac{c}{k l}\right)=1
\end{array}\right.
$$

Ta chứng minh $\operatorname{gcd}(a b, c)=1$ nếu $\operatorname{gcd}(a, b)=\operatorname{gcd}(b, c)=\operatorname{gcd}(a, c)=1$. Thật vậy, giả sử ngược lại, tức $\operatorname{gcd}(a b, c) \neq 1$. Khi đó tồn tại $p$ là ước nguyên tố chung của $a b, c$. Nhưng $a b: p$ thì ta phải có $a: p$ hoặc $b: p$, nên $\operatorname{gcd}(a, c): p$ hoặc $\operatorname{gcd}(b, c)$ : $($ cả 2 đều mâu thuẫn với giả thiết).

Áp dụng quan sát trên cho (3.1), ta được
$$
\operatorname{gcd}\left(\dfrac{a b}{k k}, \dfrac{c}{k l}\right)=1 \Leftrightarrow \operatorname{gcd}(a b, c)=k l=\operatorname{gcd}(a, c) \operatorname{gcd}(b, c)
$$

Phần 4: ta chứng minh $\operatorname{gcd}\left(a^n, b^n\right)=1$ nếu $\operatorname{gcd}(a, b)=1$. Thật vậy, giả sử $\operatorname{gcd}\left(a^n, b^n\right) \neq 1$, khi đó $a^n, b^n$ phải có một ước nguyên tố chung $p$. Sử dụng tính chất nếu $x y: p$ thì $x: p$ hoặc $y: p$. Từ đó $a, b: p$, vô lý.

Đặt $d=\operatorname{gcd}(a, b)$, ta có $\operatorname{gcd}\left(\dfrac{a}{d}, \dfrac{b}{d}\right)=1$, nên
$$
\operatorname{gcd}\left(\left(\dfrac{a}{d}\right)^n,\left(\dfrac{b}{d}\right)^n\right)=1
$$

Nhân $d^n$ cho cả 2 vế, và dùng tính chất 2 , ta được
$$
\operatorname{gcd}(a, b)^n=d^n=d^n \operatorname{gcd}\left(\left(\dfrac{a}{d}\right)^n,\left(\dfrac{b}{d}\right)^n\right)=\operatorname{gcd}\left(a^n, b^n\right)
$$

Hệ quả 3.1
Giả sử $a, b, c, n$ là các số nguyên dương, chứng minh những tính chất sau
(a) Nếu $a b: c$ và $(a, b)=1$, tồn tại $k, l$ sao cho $k l=c$, và $a: k, b \vdots l$.
(b) Nếu $a b=c^n$ và $(a, b)=1(n \geq 2)$, tồn tại $k, l$ sao cho $k l=c$ và $a=k^n, b=l^n$.

Chứng minh.

Phần 1: gọi $k=\operatorname{gcd}(a, c), l=\operatorname{gcd}(b, c)$, theo bài tập trước, ta có $k l=\operatorname{gcd}(a, c) \operatorname{gcd}(b, c)=$ $\operatorname{gcd}(a b, c)=c$, và $a: k, b: l$ theo định nghĩa.

Phần 2: gọi $k=\operatorname{gcd}(a, c), l=\operatorname{gcd}(b, c)$, theo bài tập trước, ta có $k l=\operatorname{gcd}(a, c) \operatorname{gcd}(b, c)=\operatorname{gcd}(a b, c)=$ c. Mặt khác
$$
k^n=\operatorname{gcd}\left(a^n, c^n\right)=\operatorname{gcd}\left(a^n, a b\right)=a \operatorname{gcd}\left(a^{n-1}, b\right)=a
$$
, ở đây $\operatorname{gcd}\left(a^{n-1}, b\right)=1$ do nếu tồn tại $p$ là ước nguyên tố chung cho $a^{n-1}, b$, ta phải có $p$ là ước chung của $a, b$ (vô lý). Chứng minh tương tự, ta cũng có $l^n=b$. Ta có đpcm.

B. MỘT SỐ VÍ DỤ ÁP DỤNG
Ví dụ 3.1 (Junior Balkan Mathematical Olympiad 2001).
Tìm ước chung lớn nhất của $A_0, A_1, A_2, \ldots, A_{1999}$, với $A_n=2^{3 n}+3^{6 n+2}+5^{6 n+2}$.

Hướng dẫn giải

Do $A_0=35=5 \cdot 7$, nên ước chung lớn nhất, gọi là $d$, phải là 1 trong 4 số ${1,5,7,35}$. Do $A_1=$ $2^3+3^8+5^8 \equiv 8+(-2)^8 \equiv 4(\bmod 5)$ nên $d \neq 5,35$. Mặt khác, theo định lý Fermat, ta có $3^6 \equiv 5^6$ $(\bmod 7)$, nên
$$
A_n \equiv 8^n+\left(3^6\right)^n \cdot 9+\left(5^6\right)^n \cdot 25 \equiv 1+9+25 \equiv 0 \quad(\bmod 7)
$$
Ta kết luận $d=7$.


Ví dụ 3.2. Chứng minh rằng nếu $d>0$ không phải là số chính phương, thì $\sqrt{d}$ là số vô tỷ.

Hướng dẫn giải

Để ý rằng $d=1^2 \cdot d$ nên $d$ luôn có thể viết thành dạng $d=x^2 y$ (với $x, y>0$ ). Chọn $x$ lớn nhất có thể, và để ý $y \neq 1$. Nếu $y$ có ước chính phương $z^2$ ngoài 1 , thì $d=x^{\prime 2} y^{\prime}$, với $x^{\prime}=x z>x$ và $y^{\prime}=\dfrac{y}{z}$, vô lý. Như vậy $y$ là tích các số nguyên tố khác nhau (do nếu $p$ là ước nguyên tố của $y$, thì $\dfrac{y}{p}$ không thể nào chia hết cho $p$ được).

Giả sử $\sqrt{d}=\dfrac{a}{b}$ là một số hữu tỷ, với $a, b$ nguyên dương nguyên tố cùng nhau. Ta có $$ a^2=b^2 d=(b x)^2 \cdot y $$ nên $a^2: y$. Nhưng $y$ chỉ là tích các số nguyên tố khác nhau, nên $a: y$. Thế $a=c y$ vào (*), ta được
$$
c^2 y^2=(b x)^2 y \Leftrightarrow b^2 x^2=c^2 y
$$

Để ý $c^2 y: b^2$, nhưng $(c, b)=1$ (do $(a, b)=1$ ), nên $y: b^2$. Ta đã chọn sao cho $y$ không thể nào có ước chính phương nào ngoài 1 , nên $b=1$ ! Từ đó ta có $\sqrt{d}=a$, hay $d=a^2$, vô lý.


Hướng dẫn giải

Gọi $d>0$ là một ước chung của $a^m+b^n, a^m-b^n$. Khi đó $\left\{\begin{array}{I}2 a^m=\left(a^m+b^n\right)+\left(a^m-b^n\right) \vdots d \\\ 2 b^n=\left(a^m+b^n\right)-\left(a^m-b^n\right) \vdots d\end{array}\right.$.
Để ý rằng $a, b$ khác tính chẵn lẻ, nên $a^m+b^n$ và $a^m-b^n$ luôn lẻ. Nhưng $d$ là một ước chung, nên $d$ lẻ. Như vậy $a^m, b^n: d$.

Nếu $d \neq 1$, gọi $p$ là một ước nguyên tố của $d$ (có thể $d=p$ ). Khi đó $a^m, b^n: p$, nên ta cũng có $a, b: p$. Điều này mâu thuẫn với giả thiết $a, b$ nguyên tố cùng nhau, nên $d=1$. Nhưng $d$ bất kỳ, nên $a^m+b^n, a^m-b^n$ chỉ có ước chung (dương) là 1 . Hay nói cách khác, $a^m+b^n, a^m-b^n$ nguyên tố cùng nhau.

Ví dụ 3.4. Cho 2 số hữu tỷ $\dfrac{a}{b}, \dfrac{c}{d}$ viết ở dạng tối giản (tức $(a, b)=(c, d)=1$ ) sao cho $d\frac{a}{b}+\dfrac{c}{d}$ là một số nguyên. Chứng minh rằng $|b|=|d|$.

Hướng dẫn giải

Ta có $\dfrac{a}{b}+\dfrac{c}{d}=\dfrac{a d+b c}{b d}$ là một số nguyên, nên $a d+b c: b$, hay $a d: b$. Nhưng $a, b$ nguyên tố cùng nhau, nên $d: b$.
Chứng minh tương tự với $a d+b c: d$, ta có $b: d$. Như vậy $|b|=|d|$.

Ví dụ 3.5 (Spanish Mathematical Olympiad 1996).
Giả sử $a, b$ là các số nguyên dương sao cho $\dfrac{a+1}{b}+\dfrac{b+1}{a}$ là số nguyên. Nếu $d$ là ước chung lớn nhất của $a, b$
(a) Chứng minh rằng $a+b \geq d^2$.
(b) Tìm một cặp $(a, b)$ mà $a+b=d^2$.

Hướng dẫn giải

(a) Đặt $a=d a^{\prime}, b=d b^{\prime}$, ta có
$$
\dfrac{a+1}{b}+\dfrac{b+1}{a}=\dfrac{d^2\left(a^{\prime 2}+b^{\prime 2}\right)+d\left(a^{\prime}+b^{\prime}\right)}{d^2 a^{\prime} b^{\prime}} \in \mathbb{Z}
$$
nên $\dfrac{d^2\left(a^{\prime 2}+b^{\prime 2}\right)+d\left(a^{\prime}+b^{\prime}\right)}{d^2}=a^{\prime 2}+b^{\prime 2}+\dfrac{a^{\prime}+b^{\prime}}{d}$ cūng là số nguyên. Như vậy $a^{\prime}+b^{\prime}: d$. Nhưng $a, b$ nguyên dương, nên $a^{\prime}+b^{\prime} \geq d$, hay $a+b=d\left(a^{\prime}+b^{\prime}\right) \geq d^2$.
(b) $a=3, b=6$, thì $\dfrac{a+1}{b}+\dfrac{b+1}{a}=3$ và $a+b=9=\operatorname{gcd}(a, b)^2$.

Ví dụ 3.6 (Romanian Mathematical Olympiad 2003).
Cho $n$ là một số chẵn nguyên dương. Tìm tất cả các số nguyên dương $a, b$ sao cho $a^n+b^n: a+b$.

Hướng dẫn giải

Do $n$ chẵn ta có $a^n-b^n: a^2-b^2: a+b$. Như vậy
$$
\left\{\begin{array}{l}
2 a^n=\left(a^n+b^n\right)+\left(a^n-b^n\right) \vdots a+b \\\
2 b^n=\left(a^n+b^n\right)-\left(a^n-b^n\right) \vdots a+b
\end{array}\right.
$$

Gọi $d=(a, b)$, và $a=d u, b=d v$, ta có $u, v$ nguyên tố cùng nhau và $\operatorname{gcd}(a, b)=2 d^n \operatorname{gcd}\left(u^n, v^n\right)=$ $2 d^n: d(u+v)$. Nói cách khác, $2 d^{n-1}: u+v$.

Để cho ra tất cả giá trị $a, b$ có thể, ta bắt đầu với 2 số $u, v$ nguyên dương và nguyên tố cùng nhau. Tiếp theo chọn $d$ bất kỳ sao cho $2 d^{n-1}: u+v(d$ luôn tồn tại do ta có thể chọn $d=u+v)$. Khi đó $a=d u, b=d v$ thỏa mãn đề bài.

Thật vậy, từ $a^n+b^n=d^n\left(u^n+v^n\right)$, ta chia làm 2 trường hợp
(a) Nếu $u, v$ đều lẻ: ta có $u^n+v^n$ chẵn, nên $a^n+b^n: 2 d^n: d(u+v)=a+b$.
(b) Nếu, không mất tính tổng quát, $u$ chẵn, $v$ lẻ: do $2 d^{n-1}: u+v$, và $u+v$ lẻ, nên $d^{n-1}: u+v$. Từ đó $a^n+b^n: d^n: d(u+v)=a+b$.

Ta kết luận $a=d u, b=d v$, với $u, v$ nguyên tố cùng nhau sao cho $u+v$ là ước của $2 d^{n-1}$.

Ví dụ 3.7 (India Mathematical Olympiad 1998).
Tìm tất cả các bộ số nguyên dương $(x, y, n)$ sao cho
$$
\operatorname{gcd}(x, n+1)=1 \text { và } x^n+1=y^{n+1} .
$$

Hướng dẫn giải

Do $x>0$, nên $y^{n+1}=x^n+1>1$. Ta có
$$
x^n=y^{n+1}-1=(y-1)\left(y^n+y^{n-1}+\cdots+y+1\right)
$$

Do $y-1>1$, ta phải có $y-1: p$ với $p$ là một ước nguyên tố nào đó của $x$. Từ đó
$$
y^n+y^{n-1}+\cdots+y+1 \equiv \underbrace{1+1+\cdots+1}_{n \text { số } 1} \equiv n+1 \quad(\bmod p)
$$

Như vậy $p$ là ước chung của $x$ và $n+1$, vô lý.

Ví dụ 3.8 (Bulgarian Mathematical Olympiad 2001).

Tìm tất cả các bộ $(a, b, c)$ nguyên dương sao cho $a^3+b^3+c^3$ chia hết cho $a^2 b, b^2 c$, và $c^2 a$.

Hướng dẫn giải

Đầu tiên để ý rằng nếu $d$ là ước chung của $a, b$, ta có $a^3+b^3+c^3: a^2 b: d^3$, nên $c: d$. Như vậy nếu ta đặt $d=(a, b, c)$, và $a=d u, b=d v, c=d w, u, v$ phải nguyên tố cùng nhau. Chứng minh tương tự, ta có $u, v, w$ đôi một nguyên tố cùng nhau.

Do $a^3+b^3+c^3: a^2 b$, ta có
$$
d^3\left(u^3+v^3+w^3\right): d^3 u^2 v \Leftrightarrow u^3+v^3+w^3: u^2 v
$$

Từ đó, $u^3+v^3+w^3: u^2$, và $v^3+w^3: u^2$. Chứng minh tương tự, ta cūng có $u^3+v^3+w^3: v^2, w^2$, và $w^3+u^3: v^2, u^3+v^3: w^2$. Nhưng $u, v, w$ nguyên tố cùng nhau đôi một, nên
$$
\left\{\begin{array}{l}
u^3+v^3+w^3: u^2 v^2 w^2 \\\
v^3+w^3: u^2 \\\
w^3+u^3: v^2 \\\
u^3+v^3: w^2
\end{array}\right.
$$

Không mất tính tổng quát, giả sử $u \leq v \leq w$. Do $a, b, c$ nguyên dương, $u, v, w$ cũng nguyên dương, và $u^2 v^2 w^2 \leq u^3+v^3+w^3 \leq 3 w^3$. Nói cách khác, $w \geq \dfrac{u^2 v^2}{3}$. Mặt khác, $u^3+v^3: w^2$, nên ta được
$$
u^3+v^3 \geq w^2 \geq \dfrac{u^4 v^4}{9} (*)
$$

Nhưng $u \leq v$, nên $2 v^3 \geq u^3+v^3 \geq \frac{u^4 v^4}{9}$, hay $u^4 v \leq 18$. Ta suy ra $u=1$ hoặc $u=2$. Nhưng $u=2$ thì $v \geq 2$, nên $32 \leq u^4 v \leq 18$, vô lý.
*Như vậy $u=1$. Nếu $v=1$ thì 2 : $w^2$, cho nên $w=1$. Ta có bộ $(a, b, c)=(d, d, d)$ thỏa mãn. Nếu $v \geq 2$, ta phải có $w>v$, hay $w \geq v+1 \geq 3$ do $v, w$ nguyên tố cùng nhau. Nhưng $u^3+v^3+w^3: u^2 v^2 w^2$, nên ta có
$$
1+v^3+w^3: v^2 w^2 \Rightarrow v^2 w^2 \leq 1+v^3+w^3 \leq 1+(w-1)^3+w^3<2 w^3
$$

Chia $w^2$ cho cả 2 vế, ta được $v^2<2 w$, hay $w>\frac{v^2}{2}$. Mặt khác, ta có $v^3+u^3: w^2$, nên
$$
v^3+1 \geq w^2>\frac{v^4}{4} \Leftrightarrow 4>v^3(v-4)
$$

Vậy $v \leq 4$. Nhưng $v \geq 2$, ta xét các trường hợp sau
(a) $v=4$ : khi đó $u^3+v^3=65: w^2$, nên $w=1$ (vô lý do $v \leq w$ ).
(b) $v=3$ : khi đó $u^3+v^3=28: w^2$, nên $w \in{1,2}$ (cũng vô lý như trên).
(c) $v=2$ : khi đó $u^3+v^3=9: w^2$, nên $w=3$ (do $w \geq v$ ).

Kiểm tra lại, ta nhận $(a, b, c)=(d, 2 d, 3 d)$ và các hoán vị của nó. Ta kết luận
$$
\begin{aligned}
& (a, b, c)=(k, k, k),(k, 2 k, 3 k),(k, 3 k, 2 k), \
& \quad(2 k, k, 3 k),(2 k, 3 k, k),(3 k, k, 2 k),(3 k, 2 k, k) \quad k \geq 1
\end{aligned}
$$

Ví dụ 3.9. Cho các số nguyên dương $x, y, z$ sao cho $\dfrac{1}{x}+\dfrac{1}{y}=\dfrac{1}{z}$. Giả sử $x, y, z$ nguyên tố cùng nhau (tức $(x, y, z)=1$ ), chứng minh rằng $x+y$ là một số chính phương.

Hướng dẫn giải

Viết lại phương trình thành $x+y=\dfrac{x y}{z}$. Đặt $d=(x, y)$, và $x=d a, y=d b$, ta có $(d, z)=1$ (do $(x, y, z)=1)$ và $(a, b)=1$. Thêm nữa
$$
a+b=\dfrac{d a b}{z}
$$

Ta có $d a b: z$, và $(d, z)=1$, nên $a b: z$. Do $(a, b)=1$, ta sẽ chứng minh $z$ có thể tách thành $z=r s$ sao cho $a: r$ và $b: s$.
*Đặt $r=(a, z)$ và $s=(b, z)$.

  • Theo định nghĩa $a: r, b: s$. Nhưng $(a, b)=1$, nên $(r, s)=1$.
  • Tương tự z:r,s. Kết hợp với điều chứng minh ở trên, ta có $z$ :rs.
  • Mặt khác, đặt $a=k r, b=l s$ và $z=q(r s)$, ta có $k l r s: q r s$, nên $k l: q$. Ta cũng có $(a, z)=r$, nên $(k, q s)=1$.

Như vậy $k l: q$ và $(k, q)=1$. Chứng minh tương tự, ta có $(l, q)=1$. Từ đó $q=1$, và $z=r s$.
Tóm tắt lại, ta có $a=k r, b=l s$ và $z=r s$.
*Thế vào $a+b=\dfrac{d a b}{z}$, ta có
$$
k r+l s=d k l
$$

Để ý $(a, b)=1$ nên $(k, l s)=1$. Mặt khác, $l s=d k l-k r: k$, cho nên $k=1$. Chứng minh tương tự, ta có $l=1$, nên $a b=r s=z$, và $a+b=\dfrac{d a b}{z}=d$. Từ đó $x+y=d(a+b)=d^2$ là một số chính phương.

Ví dụ 3.10. Giải phương trình nghiệm nguyên sau (theo các biến $x, y, n, m$ ) với $m, n \geq 0$.
$$
x^n+y^n=2^m
$$

Hướng dẫn giải

Đặt $d=(x, y)>0$ và $x=d u, y=d v$, ta có $u, v$ nguyên tố cùng nhau và $d^n\left(u^n+v^n\right)=2^m$. Như vậy $d=2^e\left(0 \leq e \leq \frac{m}{n}\right)$. Đặt $k=2^{m-n e}$, ta xét phương trình sau (với $u, v$ nguyên tố cùng nhau).
$$
u^n+v^n=2^k
$$
(a) Nếu $n$ chẵn

(a) Nếu $n=0$ : phương trình gốc trở thành $2^m=2$, nên $m=1$. Ta nhận bộ nghiệm $(x, y, 0,1)$ với mọi $x, y \neq 0$.

(b) Nếu $n \geq 2$ :
i. Nếu $k=0$ : ta có $u^n+v^n=1$. Nhưng $n$ chẵn, nên phương trình chỉ có 4 nghiệm $(0, \pm 1)$ và $( \pm 1,0)$. Ta nhận bộ
$$
(x, y, m, n)=\left( \pm 2^e, 0, n e, n\right),\left(0, \pm 2^e, n e, n\right) \quad(n \text { chẵn })
$$
ii. Nếu $k \geq 1$ : ta có $u^n+v^n$ chẵn. Kết hợp với $u, v$ nguyên tố cùng nhau, ta được $u, v$ cùng lẻ. Xét modulo 4, ta có $2^k=u^n+v^n \equiv 1+1 \equiv 2(\bmod 4)$. Nói cách khác $k=1$ và $u^n+v^n=2$, hay $u, v= \pm 1$. Ta nhận bộ
$$
(x, y, m, n)=\left( \pm 2^e, \pm 2^e, n e+1, n\right) \quad(n \text { chẵn })
$$

(b) Nếu $n$ lẻ: ta có $2^k=(u+v)\left(u^{n-1}-u^{n-2} v+\cdots+v^{n-1}\right)$, nên $u+v=2^s$ với $s \geq 0$.
(a) Nếu $n=1$ : ta có $u+v=2^k$, nên ta nhận các bộ sau
$$
(x, y, m, n)=\left(u, 2^m-u, m, 1\right) \quad(u \text { nguyên bất kỳ })
$$

(b) Nếu $n \geq 3$ :
i. Nếu $k=0$ : ta có $u^n+v^n=1$. Nhưng $u^n+v^n=(u+v)\left(u^{n-1}-u^{n-2} v+\cdots-u v^{n-2}+v^{n-1}\right)$, cho nên $u+v= \pm 1$.
*Với $v=1-u$, ta xét phương trình sau
$$
u^n-(u-1)^n=1
$$

Ta có $u=0,1$ là nghiệm, cho nên ta nhận các bộ sau
$$
(x, y, m, n)=\left(2^e, 0, n e, n\right),\left(0,2^e, n e, n\right) \quad(n \text { lẻ })
$$

Nếu $u \geq 2$, ta chứng minh
$$
u^n-(u-1)^n>1
$$
với mọi $n \geq 2$ bằng quy nạp. Khi $n=2$, ta có $u^2-(u-1)^2=2 u-1 \geq 3>1$. Giả sử bất đẳng thức đúng với $n$, ta chứng minh nó đúng với $n+1$
$$
\begin{aligned}
u^{n+1}-(u-1)^{n+1} & =u^n+(u-1) u^n-(u-1)^{n+1} \
& =u^n+(u-1)\left[u^n-(u-1)^n\right] \
& \geq 2^n+(2-1) \cdot 1>1
\end{aligned}
$$

Nếu $u \leq-1$, ta cũng chứng minh $u^n-(u-1)^n=(1-u)^n-(-u)^n>1$ với mọi $n \geq 2$ bằng quy nạp. Khi $n=2$, ta có $(1-u)^n-(-u)^n=-2 u+1>1$. Giả sử bất đẳng thức đúng với $n$, ta chứng minh nó đúng với $n+1$
$$
\begin{aligned}
(1-u)^{n+1}-(-u)^{n+1} & =(1+w)^{n+1}-w^{n+1} \quad(\text { đặt } w=-u \geq 1) \
& =w(w+1)^n+(w+1)^n-w^{n+1} \
& =(w+1)^n+w\left[(w+1)^n-w^n\right] \
& \geq 2^n+1 \cdot 1>1
\end{aligned}
$$
*Với $v=-1-u$, ta xét phương trình sau
$$
u^n-(u+1)^n=1 \Leftrightarrow(u+1)^n-u^n=-1
$$

Dùng những bất đẳng thức ta đã chứng minh ở trên, cộng với trường hợp $u=0,1$ không thỏa, ta kết luận trường hợp này vô nghiệm.
ii. Nếu $k \geq 1$ : ta có $u^n+v^n$ chẵn, và $u, v$ nguyên tố cùng nhau, nên $u, v$ cùng lẻ. Như vậy
$$
u^{n-1}-u^{n-2} v+\cdots-v^{n-1} \equiv \underbrace{1+1+\cdots+1}_{n \text { số } 1} \equiv n \equiv 1 \quad(\bmod 2)
$$

Kết hợp với $(u+v)\left(u^{n-1}-u^{n-2} v+\cdots+v^{n-1}\right)=2^k$, ta phải có

Kết hợp với $(u+v)\left(u^{n-1}-u^{n-2} v+\cdots+v^{n-1}\right)=2^k$, ta phải có
$$
\left\{\begin{array}{l}
u+v=2^k \quad(k \geq 1) \\\
u^{n-1}-u^{n-2} v+\cdots+v^{n-1}=1
\end{array}\right.
$$

Để ý $u^n+v^n=2^k=u+v$, ta sẽ chứng minh

$\left(u^n-v^n\right)(u-v) \geq 0$ với mọi $u, v$ và $n$ lẻ bằng quy nạp lên $n$. Trường hợp $n=1$ chính là $(u-v)^2 \geq 0$, còn $n=3$ là $\left(u^3-v^3\right)(u-v)=(u-v)^2\left(u^2+u v+v^2\right) \geq 0$.

Giả sử nó đúng với $n-2$ và $n$, ta chứng minh nó cũng đúng với $n+2$
$$
\begin{aligned}
\left(u^{n+2}-v^{n+2}\right)(u-v)= & \left(u^{n+2}-u^2 v^n+u^2 v^n-u^n v^2+u^n v^2-v^{n+2}\right)(u-v) \
= & u^2\left(u^n-v^n\right)(u-v)+u^2 v^2\left(u^{n-2}-v^{n-2}\right)(u-v) \
& +v^2\left(u^n-v^n\right)(u-v) \geq 0
\end{aligned}
$$

  • $2\left(u^n+v^n\right) \geq\left(u^2+v^2\right)\left(u^{n-2}+v^{n-2}\right)$ với mọi $u, v$ và $n \geq 3$ lẻ. Thật vậy, bất đẳng thức tương đương với
    $$
    u^n+v^n-u^2 v^{n-2}-v^2 u^{n-2} \geq 0 \Leftrightarrow\left(u^{n-2}-v^{n-2}\right)(u-v) \geq 0
    $$

Đúng theo bất đẳng thức ta đã chứng minh ở trên.
Áp dụng vào bài toán, ta có $u+v=u^n+v^n \geq \frac{u^2+v^2}{2} \cdot\left(u^{n-2}+v^{n-2}\right) \geq\left(\frac{u^2+v^2}{2}\right)^2$. $\left(u^{n-4}+v^{n-4}\right) \geq \cdots\left(\frac{u^2+v^2}{2}\right)^{(n-1) / 2}(u+v)$. Nhưng $u+v=2^k \geq 2^1>1$, nên
$$
\left(\frac{u^2+v^2}{2}\right)^{(n-1) / 2} \leq 1 \Leftrightarrow u^2+v^2 \leq 2
$$

Xét các giá trị $u, v=0, \pm 1$ thỏa mãn điều kiện trên, ta được các cặp $(u, v)=$ $(0,0),( \pm 1,0),(0, \pm 1),( \pm 1, \pm 1)$. Thử vào $u+v=u^n+v^n=2^k$ (với $2^k \geq 2^1=2$ ), ta chỉ có đúng $u=v=1$ và $k=1$ thỏa. Ta nhận các bộ
$$
(x, y, m, n)=\left(2^e, 2^e, n e+1, n\right) \quad(n \geq 3 \text { lẻ })
$$

Tổng hợp các trường hợp lại, ta kết luận các nghiệm $(x, y, m, n)$ như sau
(a) $\left(2^e, 0, n e, n\right),\left(0,2^e, n e, n\right)$, và $\left(2^e, 2^e, n e+1, n\right)(e, n \geq 0)$.
(b) $\left(-2^e, 0, n e, n\right),\left(0,-2^e, n e, n\right)$, và $\left( \pm 2^e, \pm 2^e, n e+1, n\right)(e, n \geq 0, n$ chẵ $)$.
(c) $\left(u, 2^m-u, m, 1\right)(u \in \mathbb{Z}, m \geq 0)$

Ví dụ 3.11. Tìm tất cả các số nguyên dương $x, y, z$ sao cho
$$
16 x y z=d(x+y+z)^2
$$
với $d$ là ước chung của $x, y, z$

Hướng dẫn giải

Đặt $x=d a, y=d b, z=d c$, ta có $(a, b, c)=1$. Phương trình tương đương với $16 a b c=(a+b+c)^2$.
*Gọi $p^{2 k+1}$ là một ước của $a, p$ nguyên tố. Ta sẽ chứng minh $p^{2 k+2}$ cũng là ước của $a$.

(a) Nếu $p=2$ : đặt $a=2^{2 k+1} u$, ta có
$$
2^{2 k+5} u b c=\left(2^{2 k+1} u+b+c\right)^2
$$

Nếu $b$ chẵn thì $c$ cũng phải chẵn (và ngược lại), nhưng điều này mâu thuẫn với $a, b, c$ nguyên tố cùng nhau. Như vậy $b, c$ phải lẻ. Đê ý vế trái là bội của $2^{2 k+5}$ (mũ lẻ), nên $Q^2: 2^{2 k+5}(Q=$ $2^{2 k+1} u+b+c$. Nói cách khác, $Q: 2^{k+3}$ hay $Q=2^{k+3} R$. Từ đó
$$
2^{2 k+5} u b c=Q^2=2^{2 k+6} R^2
$$
nên $2^{2 k+5} u b c: 2^{2 k+6}$. Nhưng $b, c$ lẻ, nên ta có $u: 2$. Như vậy $a=2^{2 k+1} u: 2^{2 k+2}$.
(b) Nếu $p>2$ : lập luận tương tự như trên, ta đặt $Q=a+b+c$ và $a=p^{2 k+1} u$. Phương trình tương đương với
$$
Q^2=16 p^{2 k+1} u b c: p^{2 k+1}
$$
hay $Q: p^{k+2}$. Ta có $16 p^{2 k+1} u b c=Q^2: p^{2 k+2}$. Nhưng $p>2$, nên $u b c: p$.

Giả sử, không mất tính tổng quát b:p. Khi đó $(a+b+c)^2=16 a b c: p$, nên $a+b+c: p$. Nhưng $a: p$, nên c:p. Ta có điều vô lý do $a, b, c$ nguyên tố cùng nhau. Như vậy $u: p$, nên $a=p^{2 k+1} u: p^{2 k+2}$.

Như vậy nếu $a=p_1^{\alpha_1} p_2^{\alpha_2} \cdots p_m^{\alpha_m}$ là phân tích thừa số nguyên tố, các số mũ $\alpha_i$ phải chẵn (nếu $\alpha_i$ lẻ thì $p_i^{\alpha_i+1}$ cũng là ước của $a$, vô lý). Cùng với $a>0$, ta kết luận $a$ là số chính phương. Chứng minh tương tự, $b, c$ cũng chính phương.
*Đặt tiếp $a=u^2, b=v^2, c=w^2(u, v, w>0)$, ta có phương trình
$$
16 u^2 v^2 w^2=\left(u^2+v^2+w^2\right)^2 \Leftrightarrow u^2+v^2+w^2=4 u v w
$$

Do $a, b, c$ nguyên tố cùng nhau, $u, v, w$ cũng phải nguyên tố cùng nhau. Mặt khác, xét modulo 4 cho cả 2 vế, ta có $u^2+v^2+w^2 \equiv 0,1,2,3(\bmod 4)$, với $u^2+v^2+w^2 \equiv 0(\bmod 4)$ khi và chỉ khi $u^2, v^2, w^2 \equiv 0 (\text{b mod 4} )$. Như vậy $u, v, w$ đều chẵn, vô lý.

Ta kết luận phương trình vô nghiệm.

Suy luận phản chứng (phần 2)

Phép phản chứng trong toán học còn được gọi là phương pháp chứng minh bằng mâu thuẫn. Nếu ta muốn chứng minh kết luận của bài toán là đúng thì cần phải chứng minh điều ngược lại với giả thiết là sai. Sau đây ta xét một vài ví dụ áp dụng suy luận này, dành cho các bạn hs lớp 8, 9.

1/ Ví dụ:

Ví dụ 1. 

Chứng minh rằng $\sqrt{2}$ là một số vô tỷ.

Lời giải

Giả sử $\sqrt{2}$ là số hữu tỉ. Khi đó tồn tại $a,b\in \mathbb{N}^*$ sao cho $\sqrt{2}= \dfrac{a}{b}$ với $(a,b)=1$

Ta có: $(\sqrt{2})^2=\left(\dfrac{a}{b}\right)^{2}$ hay $a^{2}=2 b^{2}\quad (1)$

Suy ra a là số chẵn, ta có: $\mathrm{a}=2 \mathrm{c}$ với $c\in Z$

Thay $\mathrm{a}=2 \mathrm{c}$ vào (1) ta được: $(2 c)^{2}=2 b^{2}$ hay $b^{2}=2 c^{2}$

Do đó, b là số chẵn

Hai số a và $b$ đều số chẵn $\Rightarrow$ Mâu thuẫn với $(1)$

Vậy $\sqrt{2}$ là số vô tỉ.

Ví dụ 2. 

Chứng minh rằng tổng của một số hữu tỷ và một số vô tỷ là số vô tỷ.

Lời giải

Giả sử tổng của số hữu tỉ a vs số vô tỉ b là số hữu tỉ c, ta có: $\mathrm{b}=\mathrm{c}-\mathrm{a}$

Mà hiệu của 2 số hữu tỉ phải là số hữu tỉ nên $b$ là số hữu tỉ

$\Rightarrow$ Mâu thuẫn vs giả thiết

Vậy tổng của 1 số hữu tỉ với 1 số vô tỉ là 1 số vô tỉ.

Ví dụ 3. (Nguyên lý Dirichlet)

Có $nk + 1$ viên bi, bỏ vào trong $k$ cái hộp. Chứng minh rằng có ít nhất một hộp có ít nhất là là $n+1$ viên bi.

Lời giải

Giả sử tất cả các hộp đều chứa số bi không vượt quá $n$ viên, khi đó tổng số bi không vượt quá $nk$, mâu thuẫn. Vậy phải có một hộp chứa nhiều hơn $n$ viên bi $\Rightarrow$ đpcm.

2/ Bài tập

Bài 1. 

Cho 15 số phân biệt thỏa mãn tổng của 8 số bất kì lớn hơn tổng của 7 số còn lại. Chứng minh tất cả các số đã cho đều dương.

Lời giải

Gọi 15 số đã cho là $a_1<a_2<a_3<\cdots <a_{15}$. Ta chỉ cần chứng minh $a_1 > 0$.

Thật vậy, giả sử $a_1 \leq 0$, khi đó $$a_1 + a_2 + \cdots + a_8 \leq a_2 + a_3 + \cdots a_8 < a_9 + \cdots a_{15}$$ (mâu thuẫn).

Vậy điều giả sử là sai, hay  $0<a_1\Rightarrow 15$ số đã cho đều dương.

Bài 2. 

Từ 8 số nguyên dương không lớn hơn 20, chứng minh rằng có thể chọn ra 3 số $x, y, z$ là độ dài 3 cạnh của một tam giác.

Lời giải

Gọi 8 số nguyên dương không lớn hơn 20 là $a_{1}, a_{2}, a_{3}, \ldots, a_{8}$

$$ \text { với } 1 \leq a_{1} \leq a_{2} \leq a_{3} \leq a_{4} \leq \ldots \ldots \leq a_{8} \leq 20 $$

Nhận thấy rằng với ba số nguyên dương $a, b, c$ thỏa mãn $a \geq b \geq c$ và $b+c>a$ thì khi đó $a, b, c$ là độ dài 3 cạnh tam giác.

Giả sử trong các số $a_{1}, a_{2}, a_{3}, a_{4}, \ldots . a_{8}$ không chọn được 3 số nào là độ dài 3 cạnh của tam giác thì ta có:

$$a 3 \geq a 1+a 2 \geq 1+1=2$$

$$a 4 \geq a 2+a 3 \geq 1+2=3$$

$$a 5 \geq a 3+a 4 \geq 2+3=5$$

$$a 6 \geq a 4+a 5 \geq 3+5=8$$

$$a 7 \geq a 5+a 6 \geq 5+8=13$$

$$a 8 \geq a 6+a 7 \geq 13+8=21$$

$\Rightarrow$ Trái với giả thiết

Vậy điều giả sử là sai

$\Rightarrow$ đpcm.

Bài 3. 

Cho tập $B = {1, 2, 3, …, 16}$. Người ta ghi các số của tập B thành một vòng tròn (mỗi số ghi một lần). Hỏi có cách ghi để tổng thỏa:

a/ Tổng của hai số kề nhau bất kì lớn hơn hoặc bằng 17 được không? Tại sao?

b/ Tổng của ba số kề nhau bất kì lớn hơn 24 được không? Tại sao?

Lời giải

a/ Giả sử tồn tại cách ghi thỏa mãn. Khi đó, gọi 2 số kề với 1 là a và b.

Theo giả thiết, ta có:

$\left\{\begin{array}{l} 1 + a \geqslant 17  \\1 + b \geqslant 17  \end{array} \right. \Rightarrow \left\{\begin{array}{l}  a \geqslant 16 \\ b \geqslant 16 \end{array} \right. \Rightarrow$ Mâu thuẫn.

Vậy không tồn tại cách ghi thỏa mãn.

b/ Giả sử tồn tại cách ghi thỏa mãn.

Khi đó, ta tách số 16 ra và chia 15 số còn lại thành 5 bộ 3 số kề nhau. Và tổng của 16 số này phải lớn hơn hoặc bằng: $16+5\cdot 25=141$

Mà $1+2+3+\cdots 16=136 \Rightarrow $ Mâu thuẫn

Vậy không tồn tại cách ghi thỏa mãn.

Bài 4. 

Có thể chia tập $X = \{1, 2, …, 2023\}$ thành hai tập rời nhau sao cho tổng các phần tử thuộc tập này bằng 2 lần tổng các phần tử thuộc tập kia?

Lời giải

Giả sử có thể chia tập $X$ thành hai tập rời nhau $A$ và $B$ sao cho tổng các phần tử thuộc A bằng 2 lần tổng các phần tử thuộc B.

Khi đó, tổng các phần tử của 2 tập hợp này phải chia hết cho 3.

Mà ta có: $1+2+3+\cdots +2023=\dfrac{2023\cdot 2024}{2}=1012\cdot 2023 \not \vdots \ 3 \Rightarrow$ Mâu thuẫn

Vậy không thể chia tập $X$ thành hai tập rời nhau $A$ và $B$ sao cho tổng các phần tử thuộc $A$ bằng 2 lần tổng các phần tử thuộc $B$.

Bài 5. 

Một bảng vuông $8 \times 8$ khuyết các ô vuông ở hai góc đối diện. Hỏi có thể phủ các ô của bảng vuông bằng các hình Domino $1 \times 2$ mà không có quân Domino nào chồng lên nhau được không? Tại sao?

Lời giải

Không có mô tả.

Giả sử có thể phủ các ô của bảng vuông bằng các hình Domino $1 \times 2$ mà không có quân Domino nào chồng lên nhau.

Mỗi quân Domino lát vào bàn cờ luôn chiếm một ô trắng và một ô đen. Do đó, để lát được phần còn lại của bàn cờ thì số ô trắng và số ô đen bằng nhau. Mà số ô màu trắng và số ô màu đen trong phần còn lại của bàn cờ không bằng nhau. Điều này mâu thuẫn.

Vậy không thể lát được phần còn lại của bàn cờ bằng các quân Domino.

TẬP HỢP – TẬP HỢP SỐ

Ví dụ 1.1. Số nguyên $A$ được tạo thành bằng các chữ viết liền nhau các số nguyên dương từ 1 đến 60 theo thứ tự từ nhỏ đến lớn: $A=123 \ldots 585960$.
(a) Hãy chỉ ra cách xóa 100 chữ số của $A$ sao cho số $A_1$ tạo bởi các chữ số còn lại là nhỏ nhất.
(b) Hãy chỉ ra cách xóa 100 chữ số của $A$ sao cho số $A_2$ tạo bởi các chữ số còn lại là lớn nhất.

Hướng dẫn giải

(a) Số $A$ có $9+2.51=111$ chữ số. Sau khi xóa 100 chữ số của $A$ ta còn 11 chữ số.
Ta có: $A=12 \ldots 10 \ldots 20 \ldots 30 \ldots 40 \ldots 50 \ldots 60$ có 6 chữ số 0 .
Để $A_1$ nhỏ nhất ta sẽ xóa sao cho $A_1$ có nhiều số 0 đứng đầu nhất.
Theo phân bố của các số 0 trong $A$ thì số $A_1$ có thể có tối đa 5 chữ số 0 đứng đầu. Còn lại 6 chữ số của $A_1$ sẽ được lấy từ dãy số sau: 51525354555657585960 .
Vậy số $A_1=00000123450$ là số nhỏ nhất cần tìm.
(b) Tương tự lập luận ở câu a)
Ta có: $A=1 \ldots 9 \ldots 19 \ldots 29 \ldots 39 \ldots 49 \ldots 5960$ có 6 chữ số 9 .
Để $A_2$ lớn nhất thì ta sẽ xóa sao cho $A_2$ có nhiều số 9 đứng đầu nhất.
Theo phân bố của các số 9 trong $A$ thì số $A_2$ có thể có tối đa 5 chữ số 9 đứng đầu. Còn lại 6 chữ số của $A_2$ sẽ được lấy từ dãy số sau: 51525354555657585960 .
Vậy số $A_2=99999785960$ là số lớn nhất cần tìm.

Ví dụ 1.2. Cho tập $A=\{1,2,3, \ldots, 9\}$.
(a) Hãy chỉ ra một cách chia tập $A$ thành 3 tập con rời nhau, có số phần tử bằng nhau và tổng các phần tử bằng nhau.
(b) Tìm tất cả cách chia trong câu a.

Hướng dẫn giải

(a) $A_1=\{1,5,9\}, A_2=\{2,6,7\}, A_3=\{3,4,8\}$ là một cách chia thỏa đề bài.
(b) Tổng các phần tử là $1+2+\cdots+9=45$ do đó mỗi tập hợp có tổng là 15 và có 3 phần tử.
Dễ thấy $1,2,3$ không cùng một tập hợp, vì nếu cùng thì phần tử còn lại sẽ lớn hơn hoặc bằng 10 (vô lý).
Giả sử $1 \in A_1, 2 \in A_2, 3 \in A_3$. hai phần tử còn lại của $A_1$ là $a, b$, ta có $a+b=14$, chỉ có thể là 6,8 hoặc 5,9.
Nếu $6,8 \in A_1$, thì hai phần tử thuộc $A_2$ tổng là 13, chỉ có thể là 4,9 .
Khi đó $5,7 \in A_3$. Ta có các kết quả $A_1=\{1,6,8\}, A_2=\{2,4,9\}, A_3=\{3,5,7\}$.
Nếu $5,9 \in A_1$, thì hai phần tử thuộc $A_2$ có tổng 13 là 6,7.
Khi đó $4,8 \in A_3$. Các kết quả là $A_1=\{1,5,9\}, A_2=\{2,6,7\}, A_3=\{3,4,8\}$.

Ví dụ 1.3. Biết rằng:
$$
A=\{1 ; a\}, B=\{a ; b ; 3\}, C=\{2 ; 4 ; c\}, D=\{a ; b ; 4\}, E=\{a ; b ; c ; e\}
$$
và biết $A \subset D ; B \subset E ; C \subset E ; D \subset E$. Tìm các phần tử $a, b, c, e$.

Hướng dẫn giải

Từ $A \subset D$, suy ra $b=1$.
Từ $B \subset E$, thì một trong hai số $c$ hoặc $e$ phải là $3(1)$.
Từ $D \subset E$ thì một trong hai số $c$ hoặc $e$ phải là $4(2)$.
Từ $C \subset E$ và (1),(2) thì $c, e$ không nhận giá trị 2 nên $a=2$ và $e=4$, suy ra $c=3$.
Vậy $a=2, b=1, c=3, e=4$.

Ví dụ 1.4. Tập hợp $M$ chứa 4 số nguyên phân biệt được gọi là tập liên kết nếu với mỗi $x \in M$ thì ít nhất một trong hai số $x-1, x+1$ thuộc $M$. Gọi $U_n$ là số tập con liên kết của tập $\{1,2, \ldots, n\}$.
(a) Tính $U_7$.
(b) Xác định giá trị nhỏ nhất của $n$ sao cho $U_n \geq 2019$.

Hướng dẫn giải

Gọi $a<b<c<d$ là 4 phần tử của một tập liên kết M.
Vì $a-1 \notin M $ nên $a+1 \in M$, suy ra $b=+1$. Vì $d-1 \in M$, suy ra $c=d-1$.
Như vậy một tập liên kết sẽ có dạng $\{a+1, d-1, d\}$, với $\{d-a>2\}$.
(a) Có 10 tập con liên kết của tập $\{1,2,3,4,5,6,7\}$ là
$$
\begin{aligned}
& \{1,2,3,4\},\{1,2,4,5\},\{1,2,5,6\},\{1,2,6,7\}, \
& \{2,3,4,5\},\{2,3,5,6\},\{2,3,6,7\}, \
& \{3,4,5,6\},\{3,4,6,7\},\{4,5,6,7\} .
\end{aligned}
$$
(b) Gọi $D=d-a+1$ là đường kính của tập $\{a, b=a+1, c=d-1, d\}$, hiển nhiên $3<D \leq$ $n-1+1=n$.
Với $D=4$ sẽ có $n-3$ tập liên kết, với $D=5$ sẽ có $n-4$ tập liên kết, …, với $D=n$ sẽ có đúng một tập liên kết. Do đó
$$
U_n=1+2+\ldots+(n-3)=\dfrac{(n-3)(n-2)}{2} .
$$
Do đó $U_n \geq 2019 \Leftrightarrow(n-3)(n-2) \geq 4038$. Như vậy giá trị nhỏ nhất của $n$ là $n=67$.

Ví dụ 1.5. Chứng minh rằng với mọi số dương $m$ thì $\dfrac{2 m}{m^2+5}$ không thể là số nguyên.

Hướng dẫn giải

Ta có $0<\dfrac{2 m}{m^2+5}<1$ nên $\dfrac{2 m}{m^2+5}$ không thể là số nguyên.

Ví dụ 1.6. (Đề tuyển sinh vào lớp 10 chuyên toán trường PTNK năm 2014) Cho 5 số tự nhiên phân biệt sao cho tổng của ba số bất kỳ trong chúng lớn hơn tổng của hai số còn lại.
(a) Chứng minh rằng tất cả 5 số đā cho đều không nhỏ hơn 5 .
(b) Tìm tất cả các bộ gồm 5 số thỏa mãn đề bài mà tồng của chúng nhỏ hơn 40 .

Hướng dẫn giải

(a) Gọi 5 số đó là $a, b, c, d, e$, do các số là phân biệt nên ta có thể giả sử $ad+e$, suy ra $a+b+c \geq d+e+1$. Suy ra $a \geq d+e+1-b-c$.
Mặt khác, do $b, c, d, e$ là số tự nhiên nên từ $d>c>b$ ta có $d \geq c+1 \geq b+2$, suy ra $d-b \geq 2$. $e>d>c$, suy ra $e-c \geq 2$.
Do đó $a \geq(d-b)+(e-c)+1 \geq 5$. Suy ra $b, c, d, e>5$.
Vậy các số đều không nhỏ hơn 5.
(b) Nếu $a \geq 6$, suy ra $b \geq 7, c \geq 8, d \geq 9, e \geq 10$, suy ra $a+b+c+d+e \geq 40$ ( vô lý),
suy ra $a<6$.
Theo câu a ta có $a=5$. Khi đó $b+c+5 \geq d+e+1$, suy ra $b+c \geq d+e-4$.
Mà $d-2 \geq b, e-2 \geq c$, suy ra $d+e-4 \geq b+c$. Do đó $b=d-2, c=e-2$.
Khi đó $a+b+c+d+e=5+2 b+2 c+4<40$. Suy ra $b+c<\dfrac{31}{2}$. Suy ra $b \geq 7$.
Từ đó ta có $b=6, b=7$.
Nếu $b=6$ ta có $d=8, c=8, e=10$. Ta có bộ $(5,6,7,8,9)$
Nếu $b=7, d=9, c=8, e=10$.
Ta có bộ $(5,7,8,9,10)$. Vậy có hai bộ số thỏa đề bài là $(5,6,7,8,9)$ và $(5,7,8,9,10)$.

Ví dụ 1.7. Trong một buôn của người dân tộc, cư dân có thể nói được tiếng dân tộc, có thể nói được tiếng Kinh hoặc nói được cả hai thứ tiếng. Kết quả của một đợt điều tra cơ bản cho biết:
Có 912 người nói tiếng dân tộc,
Có 653 người nói tiếng Kinh,
Có 435 người nói được cả hai thứ tiếng.
Hỏi buôn làng có bao nhiêu cư dân ?

Hướng dẫn giải

Gọi $A$ là tập các người các người nói tiếng dân tộc, ta có $|A|=912, B$ là tập các người nói tiếng Kinh, ta có $|B|=653$. Khi đó $|A \cap B|=435$.
$A \cup B$ là tập các người dân trong buông.
Ta có
$$
|A \cup B|=|A|+|B|-|A \cap B|=912+653-435=1130
$$

Bài 1.1. Viết các số từ 1 đến 9 vào một bảng vuông $3 \times 3$, mỗi số viết một lần, sao cho tồng số ở mỗi dòng, mỗi cột và hai đường chéo đều được số chia hết cho 9 .
(a) Chỉ ra một cách viết thỏa đề bài.
(b) Với cách viết thỏa đề bài thì ô chính giữa có thể là các số nào? Tại sao?

Hướng dẫn giải

(a)
(b) Giả sử ta có bảng sau thỏa đề bài

Ta có $a+e+k, c+e+g, d+e+f, b+e+h$ chia hết cho 9 .

$$
a+e+k+c+e+g+d+e+f+b+e+h=3 e+a+b+c+d+e+f+g+h+k=3 e+45
$$
nên $3 e+45$ chia hết cho 9 , do dó $e$ chia hết cho 3 , vậy $e \in\{3,6,9\}$.

Bài 1.2. Tích của $n$ số nguyên bằng 1 và tổng của chúng bằng 0 . Chứng minh rằng $n$ là một số chia hết cho 4 .

Hướng dẫn giải

Gọi $n$ số đó là $a_1, a_2, \cdots, a_n$. Ta có
$$
a_1+a_2+\cdots+a_n=0
$$

$$
a_1 \cdot a_2 \cdots a_n=1
$$
nên các số $a_i \in\{-1 ; 1\}$, mà tổng bằng 0 nên số các số 1 bằng số các số -1 , do đó $n$ chẵn, đặt $n=2 k$, khi đó
$$
1=a_1 \cdot a_2 \cdots a_n=(-1)^k
$$
Do đó $k$ cũng chẵn, suy ra $n$ chia hết cho 4.

Bài 1.3. Tập hợp $\mathrm{A}$ bao gồm các số tự nhiên thỏa các điều kiện sau:
(a) $1 \in A$;
(b) Nếu $n \in A$ thì $2 n+1 \in A$;
(c) Nếu $3 n+1 \in A$ thì $n \in A$;
Vậy 8 có thuộc $A$ không ?

Hướng dẫn giải

$\{1,3,7,15,31,63,127\} \in A$, và $\{42,85,171,343,114,229,76,25,8\} \in A$

Bài 1.4. Giả sử $x, y, z, t$ là bốn số khác nhau và là các phần tử của tập hợp
$$
A=\{1 ; 2 ; 3 ; 4\} .
$$
Tìm $x, y, z, t$ với các giả thiết:
Nếu $x \neq 1$ thì $z \neq 2$;
Nếu $t=2$ thì $y \neq 1$;
Nếu $y=2$ hoặc $y=3$ thì $x=1$;
Nếu $y \neq 3$ thì $z=4$;
Nếu $t \neq 1$ thì $y=1$.

Hướng dẫn giải

Bài 1.5. Một nhóm 6 học sinh làm bài kiểm tra môn toán được điểm là số tự nhiên từ 1 đến 10 . Hai bạn được gọi là bạn tốt nếu điểm trung bình của 2 bạn đó lớn điểm trung bình của 6 bạn.
(a) Có thể chia 6 bạn thành 3 cặp bạn tốt được không? Tại sao?
(b) Nếu số điểm của 6 bạn là khác nhau, chứng minh rằng có 2 bạn có số điểm hơn kém nhau là 1 .

Hướng dẫn giải

Gọi số điểm các bạn lằn lượt là $a_1, a_2, a_3, a_4, a_5, a_6$, và $a_i \in\{1,2,3,4,5,6,7,8,9,10\}$.
Đặt $s=a_1+a_2+a_3+a_4+a_5+a_6$
(a) Giả sử chia được thành 3 cặp bạn tốt, giả sử là các cặp $a_1, a_2 ; a_3, a_4$ và $a_5, a_6$ ta có
$$
\dfrac{a_1+a_2}{2}>\dfrac{s}{6}, \dfrac{a_3+a_4}{2}>\dfrac{s}{6}, \dfrac{a_5+a_6}{2}>\dfrac{s}{6}
$$
Suy ra
$$
\dfrac{a_1+a_2+a_3+a_4+a_5+a_6}{2}>\dfrac{s}{2}
$$

Điều này mâu thuẫn.
(b) Giả sử không có bạn nào hơn kém nhau là 1 , thì giả sử $a_1<a_2<a_3<a_4<a_5<a_6$ Suy ra $a_2 \geq 3, a_3 \geq 5, \cdots, a_6 \geq 11$, vô lí.

Bài 1.6. Trong kỳ thi tốt nghiệp THPT ở một trường, kết quả số thí sinh đạt danh hiệu xuất sắc nhu sau:
Về môn Toán: 48 thí sinh,
Về Toán hoặc Văn: 76 thí sinh,
Về Vật lí: 37 thí sinh,
Về Văn: 42 thí sinh,
Về Vật lí hoặc Văn: 66 thí sinh,
Về Toán hoặc Vật lí: 75 thí sinh,
Về cả ba môn: 4 thí sinh.
Vậy có bao nhiêu học sinh chỉ nhận được danh hiệu xuất sắc về:
(a) 1 môn ?
(b) 2 môn?
(c) Ít nhất 1 môn?

Hướng dẫn giải

Sử dụng biểu đồ Venn. Kí hiệu $A, B, C$ là tập hợp các học sinh đạt danh hiệu xuất sắc tương ứng với các môn Toán, Vật lí hoặc Văn. Các tập hợp này, theo giả thiết thì có 48,37 và 42 phần tử. Giao của ba tập hợp này có 3 phần tử. Kí hiệu qua $\mathrm{a}, \mathrm{b}, \mathrm{c}, \mathrm{x}, \mathrm{y}, \mathrm{z}$ là số các thí sinh đạt danh hiệu xuất sắc.

Theo 1,2 hoặc 3 môn. Dựa vào biểu đồ Venn ta lập được các phương trình:
$$
\left\{\begin{array}{l}
a+x+y=44 \\\
b+x+z=33 \\\
a+b+x+y+z=71 \\\
a+c+x+y+z=72 \\\\
b+c+x+y+z=62
\end{array}\right.
$$
Ta có được một hệ 6 phương trình với 6 ần, nhưng diều mà ta cần biết không phải là các giá trị ẩn $\mathrm{a}, \mathrm{b}, \mathrm{c}, \mathrm{x}, \mathrm{y}, \mathrm{z}$ mà là các tổng $\mathrm{a}+\mathrm{b}+\mathrm{c}, \mathrm{x}+\mathrm{y}+\mathrm{z}$.
Muốn vậy, ta cộng ba phương trình đầu của hệ và sau đó cộng ba phương trình sau của hệ với nhau và được:
$$
\left\{\begin{array}{l}
a+b+c+2(x+y+z)=115 \\\
2(a+b+c)+3(x+y+z)=205
\end{array}\right.
$$
Xem hệ này như là một hệ phương trình hai ẩn, ta tính được:

$$
\begin{aligned}
& a+b+c=65 \
& x+y+z=25
\end{aligned}
$$

Đáp số: 65 thí sinh đạt danh hiệu xuất sắc 1 môn, 25 thí sinh đạt danh hiệu xuất sắc 2 môn, 94 thí sinh đạt danh hiệu xuất sắc ít nhất 1 môn.

Bài 1.7. Một số $m$ được gọi là số ma thuật nếu tổng các chữ số của nó bằng tích các chữ số của nó. Ví dụ số 213 ta có $2+1+3=2 \times 1 \times 3$.
(a) Chứng minh rằng có số ma thuật có $1,2,3,4,5$ chữ số.
(b) Có số ma thuật có 6 chữ số hay không? Tại sao?
(c) Chứng minh rằng có số ma thuật có 2037 chữ số.

Hướng dẫn giải

(a) Các số ma thuật có $1,2,3,4,5$ chữ số là: $1,22,123,4211,52111$.
(b) Số ma thuật có 6 chữ số: 621111
(c) $22222222222111 \ldots .1,11$ chữ số 2 và 2025 chữ số 1 .

Bài 1.8. Có thể viết các số tự nhiên từ 1 đến 16 thành
(a) một đường thẳng
(b) một đường tròn
sao cho tồng hai số liên tiếp là bình phương của một số tự nhiên dược không? Tại sao

Hướng dẫn giải

(a) $8,1,15,10,6,3,13,12,4,5,11,14,2,7,9,16$.
(b) Giả sử tồn tại cách ghi thỏa đề bài, ta xét hai số kề bên số 8 , gọi là $a, b$ thì $8+a, 8+b$ đều là số chính phương, suy ra $a=b=1$, vô lí. Vậy không tồn tại cách ghi thỏa đề bài.

Bài 1.9. Cho $A$ là tập con của tập các số hữu tỷ dương thỏa mãn các điều kiện sau:
$1 \in A$
Nếu $x \in A$ thì $1+x \in A$
Nếu $x \in A$ thì $\dfrac{1}{x} \in A$

Hướng dẫn giải

(c) $\dfrac{13}{5}=2+\dfrac{3}{5}$.
Ta có $\dfrac{3}{5}=\dfrac{1}{1+\dfrac{2}{3}} \dfrac{3}{2} \in A \Rightarrow \dfrac{2}{3} \in A \Rightarrow \dfrac{5}{3}=1+\dfrac{2}{3} \in A$, do đó $\dfrac{3}{5} \in A$, hơn nữa $2 \in A$, suy ra $\dfrac{13}{5}=2+\dfrac{3}{5} \in A$.

Bài 1.10. Trên bảng có ghi các số tự nhiên từ 1 đến $n$. Cứ mỗi lần một học sinh xóa đi hai số và thay bằng tổng hoặc hiệu của hai số đó.
(a) Cho $n=8$ hỏi sau 7 lần có thể số trên bảng còn lại số 0 dược không?
(b) Câu hỏi tương tự với $n=9$.

Hướng dẫn giải

(a) Câu trả lời là thực hiện được, ta làm như sau:
$1,2,3,4,5,6,7,8$
$1,2,3,4,5,6,1$
$1,2,3,4,1,1$
$1,2,1,1,1$
$1,1,1,1$,
$1,1,0$
$0,0$
$0$
(b) Câu trả lời là không, vì mổi lần thay đổi thì tổng các số còn lại tính chẵn lẻ khồng đổi, tổng lúc đầu là $1+2+\cdots+9=45$ nên sau một số lần thay đổi thì số còn lại phải là số lẻ, không thể bằng 0 .

Bài 1.11. Có bao nhiêu cách viết số 1 thành tồng của 3 phân số mà mỗi phân số có tử số bằng 1 và mẫu số là một số tự nhiên? Tại sao?

Hướng dẫn giải

$$
1=\dfrac{1}{6}+\dfrac{1}{3}+\dfrac{1}{2}=\dfrac{1}{4}+\dfrac{1}{4}+\dfrac{1}{2}=\dfrac{1}{3}+\dfrac{1}{3}+\dfrac{1}{3}
$$

Bài 1.12. Chứng minh rằng giữa hai số hữu tỉ phân biệt luôn có một số hữu tỉ.

Hướng dẫn giải

Cho $a, b \in \mathbb{Q}, a<b$. Xét $c=\frac{a+b}{2}$ ta có $a<c<b$ và $c \in \mathbb{Q}$.

Bài 1.13. Gọi $S$ là tập hợp các số tự nhiên có thể viết thành tổng bình phương của hai số tự nhiên khác, ví dụ $5=1^2+2^2$ thì $5 \in S$. Chứng minh rằng nếu $x, y \in S$ thì $x y \in S$.

Hướng dẫn giải

Cho $a, b \in S$ ta có $a=x^2+y^2, b=z^2+t^2$, khi đó
$$
a b=\left(x^2+y^2\right)\left(z^2+t^2\right)=x^2 z^2+y^2 t^2+x^2 t^2+y^2 z^2=(x z+t y)^2+(x z-t y)^2
$$
Do đó $a b \in S$.

Bài 1.14. Cho $a, b$ là các số nguyên dương phân biệt, chứng minh rằng 1 không là nghiệm của phương trình $x^2-2(a+b) x+a b+2=0$.

Hướng dẫn giải

Giả sử 1 là nghiệm của phương trình ta có
$$
1^2-2(a+b) 1+a b+2=0 \Leftrightarrow a b-2 a-2 b+3=0 \Leftrightarrow(a-2)(b-2)=1
$$
Do $a, b$ là các số nguyên dương nên $a=1, b=1$ hoặc $a=3, b=3$ mâu thuẫn vì $a \neq b$.

Bài 1.15. Cho các số $a_1, a_2, \cdots, a_6$ thỏa $-\dfrac{1}{2} \leq a_i \leq \dfrac{1}{2}$ và tổng của 5 số bất kì là một số nguyên. Chứng minh rằng 6 số này bằng nhau.

Hướng dẫn giải

Đặt $S=a_1+a_2+\cdots a_6$, ta có $S \in \mathbb{Z}$
Ta có $S-a_i \in \mathbb{Z}$ với mọi $i$.
Giả sử có hai số $a_1 \neq a_2$ ta có $S-a_1-\left(S-a_2\right) \in \mathbb{Z} \Rightarrow a_2-a_1 \in \mathbb{Z}$, suy ra $a_1, a_2 \in\{\dfrac{1}{2},-\dfrac{1}{2}\}$, do $a_1 \neq a_2$ nên $a_1=\dfrac{1}{2}, a_2=-\dfrac{1}{2}$ hoặc $a_1=\dfrac{-1}{2}, a_2=\dfrac{1}{2}$.
Tương tự xét cặp số giữa $a_1$ với các số $a_3,a_4, a_5, a_6$ ta có cũng có các số còn lại thuộc $\{\dfrac{1}{2}, \dfrac{-1}{2}\}$, do đó tổng 5 số lúc này không thể là số nguyên.

ĐỀ VÀ ĐÁP ÁN THI VÀO 10 CHUYÊN TOÁN SGD THÀNH PHỐ HỒ CHÍ MINH NĂM 2023

THỜI GIAN LÀM BÀI 150 PHÚT

Bài 1. (1,0 diểm) Cho $a, b$ là các số thực, $b \neq 0$ thỏa mãn điều kiện
$$
a^2+b^2=\frac{4 b^2}{\sqrt{a^2+b^2}+a}+a \sqrt{a^2+b^2}
$$

Tính giá trị của biểu thức $P=a^2+b^2$.
Bài 2. (2,5 điếm)
a) Giải phương trình: $x=\frac{5}{x-1}+2 \sqrt{x-2}$.
b) Giải hệ phương trình $\left\{\begin{array}{l}\frac{9 y+49}{x+y}+x+y=23 \\\ x \sqrt{x}+y \sqrt{y}=7(\sqrt{x}+\sqrt{y})\end{array}\right.$.

Bài 3. (2,5 điểm) Cho tam giác $A B C$ vuông tại $A(A B<A C)$, có đường cao $A H$. Dường tròn tâm $I$ nội tiếp tam giác $A B C$, tiếp xúc với các cạnh $B C, C A, A B$ lần lượt tại $D, E, F$. Gọi $J$ là giao điểm của $A I$ và $D E . K$ là trung điểm $A B$.
a) Chứng minh tứ giác $B I J D$ nội tiếp
b) Gọi $M$ là giao điểm của $K I$ và $A C, N$ là giao điểm của $A H$ và $E D$. Chứng minh $A M=A N$.
c) Gọi $Q$ là giao điểm của $D I$ và $E F, P$ là trung điểm của $B C$. Chứng minh ba điểm $A, P, Q$ thẳng hàng.

Bài 4. (2,0 diểm) Cho các số thực dương $x, y, z$ thỏa mãn $\sqrt{1+4 x y+2 x+2 y}+2 z=5$.
a) Chứng minh $\frac{1}{\sqrt{(2 x+1)(2 y+1)}}+\frac{1}{2 z+1} \geq \frac{2}{3}$.
b) Tìm giá trị nhỏ nhất của biễu thức $P=\frac{x+1}{2 x+1}+\frac{y+1}{2 y+1}+\frac{2 z+3}{4 z+2}$.

Bài 5. (1,0 điểm) Cho đường tròn tâm $O$ nội tiếp hình thoi $A B C D$. Gọi $E, F, G, H$ là các điểm lần lượt thuộc các cạnh $A B, B C, C D, D A$ sao cho $E F, G H$ cùng tiếp xúc với $(O)$.
a) Chứng minh $C G \cdot A H=A O^2$.
b) Chứng minh $E H$ song song $F G$.

Bài 6. (1,0 điểm) Xét các số nguyên $a<b<c$ thỏa mãn $n=a^3+b^3+c^3-3 a b c$ là số nguyên tố.
a) Chứng minh $a<0$.
b) Tìm tât cả các số nguyên $a, b, c(a<b<c)$ sao cho $n$ là một ước của 2023.

ĐÁP ÁN CỦA GIÁO VIÊN STAR EDUCATION

ĐỀ VÀ ĐÁP ÁN THI VÀO 10 CHUYÊN TOÁN TRƯỜNG PHỔ THÔNG NĂNG KHIẾU NĂM 2024

Thời gian làm bài: 150 phút

Đề bài:

Bài 1. (2 điểm)
1) Giải hệ phương trình $\left\{\begin{array}{l}x^3+z^3=y \\\ y^3+x^3=z \\\ z^3+y^3=x\end{array}\right.$.
2) Cho hai số nguyên dương $a, b$ phân biệt. Chứng minh phương trình sau có đúng ba nghiệm
$$
(\sqrt{x}-1)\left[x^2-2(a+b) x+a b+2\right]=0 .
$$

Bài 2. (1.5 điểm) Cho ba số thực $a, b, c$ không âm thóa mãn: $a^2+b^2+c^2+3=2(a b+b c+c a)$.
Chứng minh
$$
3 \leq a+b+c \leq \frac{2(a b+b c+c a)+3}{3} .
$$

Bài 3. (2 điểm) Với mỗi số tự nhiên $\mathrm{n}$, đặt $a_n=(2+\sqrt{3})^n+(2-\sqrt{3})^n$.
a) Chứng minh $a_{n+2}=4 a_{n+1}-a_n$ với mọi $n=0,1,2, \ldots$.
b) Tìm $\mathrm{n}$ để $a_n$ chia hết cho 4 .
c) Tìm $\mathrm{n}$ đề $a_n$ chia hết cho 14 .

Bài 4. (3 điểm) Cho tứ giác $A B C D$ nội tiếp đường tròn $(\mathrm{O})$ có tam giác $A B D$ là tam giác nhọn và đường chéo $\mathrm{AC}$ đi qua tâm $\mathrm{O}$ của đường tròn $(\mathrm{O})$. Gọi $\mathrm{I}$ là trung điểm $\mathrm{BD}, \mathrm{H}$ là trực tâm của tam giác $A B D$, $\mathrm{E}$ là giao điểm khác $\mathrm{A}$ của $\mathrm{AI}$ với $(\mathrm{O})$ và $\mathrm{K}$ là hình chiếu vuông góc của $\mathrm{H}$ lên $\mathrm{AI}$.
a) Chứng minh $C E H K$ là hình bình hành và $I B^2=I D^2=I A \cdot I K$.
b) Lấy điểm $\mathrm{F}$ trên cung nhỏ $\widehat{B D}$ của đường tròn $(\mathrm{O})$ sao cho $\widehat{B A F}=\widehat{D A I}$. Chứng minh các điểm $\mathrm{K}$ và $\mathrm{F}$ đối xứng nhau qua đường thẳng $\mathrm{BD}$.
c) Chứng minh các đường phân giác trong các góc $\widehat{B A D}$ và $\widehat{B K D}$ cắt nhau trên $\mathrm{BD}$.
d) Trên đường thẳng qua $\mathrm{H}$ và song song $\mathrm{AC}$ lấy điểm $\mathrm{T}$ sao cho $T H=T K$. Chứng minh các điểm $\mathrm{O}, \mathrm{K}, \mathrm{F}, \mathrm{T}$ cùng thuộc một đường tròn.

Bài 5. (1.5 điểm) Cho các sổ nguyên dương $a_1<a_2<a_3<\ldots<a_{30}<a_{31}$. Người ta ghi tất cả các số này lên 31 chiếc thẻ, mỗi thẻ ghi một số.
a) Biết rằng tổng các số được ghi trên 16 thẻ bất kỳ trong số 31 thẻ trên luôn lớn hơn tổng các số được ghi trên 15 thè còn lại. Chứng minh $a_1 \geq 226$.
b) Lấy $a_1, a_2, \ldots, a_{31}$ là 31 số nguyên dương đầu tiên: $1,2, \ldots, 31$. Người ta bỏ 31 thẻ được ghi các số này vào hai chiếc hộp một cách ngẫu nhiên. Khi kiểm tra một hộp thi thấy rằng trong hộp đó không có hai thẻ nào có tồng hai số được ghi là số chính phương. Chứng minh trong hộp còn lại ta có thể chọn ra được bốn thè và chia chúng thành hai cặp sao cho tổng hai sô̂ được ghi trên mỗi cặp là số chính phương.

Đáp án tham khảo từ Star Education

Bài toán hàm số trong kì thi tuyển sinh vào 10

Trong các kì thi tuyển sinh vào 10 có dạng toán liên quan đến hàm số, chủ yếu là hàm bậc hai dạng $y = ax^2$ (1) và đường thẳng $y = mx + n$ (2)Trong bài viết này chủ yếu xét các bài toán tương giao giữa đồ thị hàm số (1) và (2).

Nếu hàm số $y =ax^2$ có đồ thị là parabol $(P)$ và hàm số $y = mx + n$ có đồ thị là đường thẳng $d$, thì phương trình hoành độ giao điểm của $(P)$ và $(d)$ là

$$ax^2 = mx + n \Leftrightarrow ax^2 – m x – n =0 (*)$$

$(*)$ là một phương trình bậc hai, nên có 3 trường hợp xảy ra:

  • TH1: Nếu $(*)$ vô nghiệm thì $(d)$ và $(P)$ không có giao điểm.
  • TH2: Nếu $(*)$ có 1 nghiệm thì $(d)$ và $(P)$ có 1 giao điểm, ta nói $d$ tiếp xúc với $(P)$.
  • TH3: Nếu $(*)$ có hai nghiệm phân biệt thì ta nói $(d)$ cắt $(P)$, và nghiệm của $(*)$ là hoành độ của hai giao điểm, từ hoành độ ta có thể tính tung độ của giao điểm dựa vào phương trình của $(d)$ hoặc của $(P)$.

Ta xét một vài ví dụ sau:

Bài 1. (Thi vào lớp 10 trường PTNK năm 2018) Gọi $(P),(d)$ lần lượt là đồ thị của các hàm số $y=x^2$ và $y=2 m x+3$.
a) Chứng minh đường thẳng $(d)$ luôn cắt $(P)$ tại hai điểm phân biệt $A\left(x_1 ; y_1\right), B\left(x_2 ; y_2\right)$ và tính $y_1+y_2$ theo $m$.
b) Tìm $m$ sao cho $y_1-4 y_2=x_1-4 x_2+3 x_1 x_2$.

Lời giải bài 1.
a) Phương trình hoành độ giao điểm của $(P)$ và $(d)$ là:
$$
x^2=2 m x+3 \Leftrightarrow x^2-2 m x-3=0 \quad(1)
$$

Xét phương trình (1), ta có: $\Delta^{\prime}=m^2+3>0$ với mọi $m \in \mathbb{R}$
Suy ra phương trình (1) luôn có hai nghiệm phân biệt $x_1, x_2$ với mọi $m$ hay $(d)$ luôn cắt $(P)$ tại hai điểm phân biệt $A\left(x_1 ; y_1\right), B\left(x_2 ; y_2\right)$.
Theo định lý Viete, ta có: $\left\{\begin{array}{l}x_1+x_2=2 m \\\ x_1 x_2=-3\end{array}\right.$
Khi đó $y_1=2 m x_1+3, y_2=2 m x_2+3$
$y_1+y_2=2 m x_1+3+2 m x_2+3=2 m\left(x_1+x_2\right)+6=4 m^2+6$
b) Ta có:
$y_1-4 y_2=x_1-4 x_2+3 x_1 x_2 $
$\Leftrightarrow 2 m x_1+3-8 m x_2-12=x_1-4 x_2-9 $
$ \Leftrightarrow 2 m\left(x_1-4 x_2\right)=x_1-4 x_2 $
$ \Leftrightarrow\left(x_1-4 x_2\right)(2 m-1)=0 $
$ \Leftrightarrow\left[\begin{array}{l}
x_1=4 x_2 \\\
m=\frac{1}{2} \quad(n)
\end{array}\right. $
Với $x_1=4 x_2 $ lại có $x_1 x_2=-3 \Rightarrow 4 x_2^2=-3 $ (vô lý)
Vậy $m=\frac{1}{2} $

Bài 2. (Đề thi vào 10 trường PTNK năm 2019) Cho $(P),(d)$ lần lượt là đồ thị hàm số $y=x^2$ và $y=2 x+m$.
a) Tìm $m$ sao cho $(P)$ cắt $(d)$ tại hai điểm phân biệt $A\left(x_1 ; y_1\right), B\left(x_2 ; y_2\right)$.
b) Tìm $m$ sao cho $\left(x_1-x_2\right)^2+\left(y_1-y_2\right)^2=5$.

Lời giải bài 2.
a) Phương trình hoành độ giao điểm của $(P)$ và $(d)$
$$
x^2=2 x+m \Leftrightarrow x^2-2 x-m=0 \quad(1)
$$
$(P)$ cắt $(d)$ tại 2 điểm phân biệt $A, B \Leftrightarrow (1)$ có 2 nghiệm phân biệt

$\Leftrightarrow \Delta^{\prime}>0 \Leftrightarrow 1+m>0 $
$ \Leftrightarrow m>-1(*)$
Vậy $m>-1$ thì $(P)$ cắt $(d)$ tại hai điểm phân biệt.


b) Với điều kiện $(*)$ theo Viete ta có: $S=x_1+x_2=2, P=x_1 \cdot x_2=-m$

Ta có: $A\left(x_1 ; y_1\right) \in(d) \Leftrightarrow y_1=2 x_1+m ; B\left(x_2 ; y_2\right) \in(d) \Leftrightarrow y_2=2 x_2+m$

Ta có: $\left(x_1-x_2\right)^2+\left(y_1-y_2\right)^2=5 $

$\Leftrightarrow\left(x_1-x_2\right)^2+\left(2 x_1-2 x_2\right)^2=5$

$\Leftrightarrow\left(x_1-x_2\right)^2+4\left(x_1-x_2\right)^2=5 $

$\Leftrightarrow\left(x_1-x_2\right)^2=1 \Leftrightarrow\left(x_1+x_2\right)^2-4 x_1 x_2=1$

$\Leftrightarrow 4+4 m=1 \Leftrightarrow m=\frac{-3}{4} $ thỏa (*)
Vậy $m = \dfrac{-3}{4}$.

Bài 3. Đồ thị của hàm số $f(x)=a x^2$ và $g(x)=-a x+b(a ; b$ là các số thực), điểm chung thứ nhất có hoành độ bằng 1 và tung độ điểm chung thứ 2 là 8 . Tìm hoành độ của điểm chung thứ hai của hai đồ thị và tính $a, b$.

Lời giải bài 3.

  • Phương trình hoành độ giao điểm $a x^2=-a x+b \Leftrightarrow a x^2+a x-b=0$ thì phương trình nhận 1 là nghiệm nên $a 1^2+a \cdot 1-b=0 \Rightarrow b=2 a$.
  • Khi đó gọi nghiệm còn lại là $x_2$ ta có $1 \cdot x_2=\frac{-b}{a}=-2$
  • Do đó tung độ $a(-2)^2=8$, suy ra $a=2$ và $b=4$.

Bài 4. (TS chuyên Đăk Lăk 2020 – 2021) Trong mặt phẳng $O x y$, cho parabol $(P): y=x^2$ và đường thẳng $(d): y=2(m+1) x+3$ với $m$ là tham số. Tìm tất cả các giá trị của tham số $m$ để đường thẳng $(d)$ cắt parabol tại hai điểm phân biệt có hoành độ $x_1, x_2$ thoả mãn điều kiện $x_1^2-2 m x_1+2 x_2-x_1 x_2=2$.

Lời giải bài 4.

  • Phương trình hoành độ giao điểm $x^2-2(m+1) x-3=0\left(^*\right)$ $\Delta^{\prime}=(m+1)^2+3>0$ với mọi $m$.
  • Theo định lý Viete ta có $x_1+x_2=2(m+1), x_1 x_2=-3$.
    Ta có $x_1^2-2(m+1) x_1-3=0$, suy ra $x_1^2-2 m x_1=2 x_1+3$ $x_1^2-2 m x_1+2 x_2-x_1 x_2=2 \Leftrightarrow 2 x_1+3+2 x_2-(-3)=2 \Leftrightarrow m=-2$.
  • Vậy $m=-2$.

Bài 5. (TS chuyên Khánh Hoà 2020 – 2021) Trên mặt phẳng toạ độ $O x y$, cho parabol $(P)$ có phương trình $y=2 x^2$ và đường thẳng $(d): y=-2 m x+m+1$ với $m$ là tham số.
a) Chứng minh đường thẳng $(d)$ luôn cắt Parabol $(P)$ tại hai điểm phân biệt.
b) Gọi $x_1, x_2$ lần lượt là hoành độ giao điểm của đường thẳng $(d)$ và parabol $(P)$, tìm $m$ thoả mãn đẳng thức $\frac{1}{\left(2 x_1-1\right)^2}+\frac{1}{\left(2 x_2-1\right)^2}=66$.

Lời giải bài 5 .
a) Phương trình hoành độ giao điểm của $d$ và $P$ là
$$
2 x^2+2 m x-m-1=0
$$
$\Delta^{\prime}=m^2-2(-m-1)=(m+1)^2+1>0$ với mọi $m$, do đó $d$ cắt $P$ tại hai điểm phân biệt với mọi $m$.
b) Theo định lý Viete ta có $x_1+x_2=-m, x_1 x_2=\frac{-m-1}{2}$.
Suy ra $x_1^2+x_2^2=\left(x_1+x_2\right)^2-2 x_1 x_2=m^2+m+1$
Ta có $66=\frac{1}{\left(2 x_1-1\right)^2}+\frac{1}{\left(2 x_2-1\right)^2}=\frac{\left(2 x_1-1\right)^2+\left(2 x_2-1\right)^2}{\left(2 x_1-1\right)^2\left(2 x_2-1\right)^2}=\frac{4\left(x_1^2+x_2^2\right)-4\left(x_1+x_2\right)+2}{\left(4 x_1 x_2-2\left(x_1+x_2\right)+1\right)^2}$
$$
=\frac{4\left(m^2+m+1\right)-4(-m)+2}{(-2 m-2-2(-m)+1)^2}=\frac{4 m^2+8 m+6}{1}
$$

Giải ra được $m=-5, m=3$.

Bài 6. (TS chuyên Thái Bình 2020 – 2021) Trong mặt phẳng toạ độ $O x y$, cho parabol $(P): y=\frac{x^2}{2}$ và hai đường thẳng $\left(d_1\right): y=5 x+2,\left(d_2\right): y=\left(m^2+1\right) x+m$ với $m$ là tham số.
a) Tìm $m$ để $\left(d_1\right)$ song song với $\left(d_2\right)$.
b) Tìm $m$ để $\left(d_2\right)$ cắt parabol $(P)$ tại hai điểm phân biệt có hoành độ $x_1, x_2$ sao cho $Q=x_1+x_2-4 x_1 x_2$ đạt giá trị nhỏ nhất.

Lời giải bài 6 .
a) Điều kiện để $d_1 || d_2$ là $m^2+1=5, m \neq 2$, giải ra được $m=-2$.
b) Phương trình hoành độ giao điểm của $d_2$ và $P$ là
$$
\frac{x^2}{2}=\left(m^2+1\right) x+m \Leftrightarrow x^2-2\left(m^2+1\right) x-2 m=0
$$

Điều kiện $\Delta^{\prime}=\left(m^2+1\right)^2-(-2 m)>0 \Leftrightarrow m^4+2 m^2+1+2 m>0 \Leftrightarrow m^4+m^2+(m+1)^2>0$ (Đúng với mọi $m)$

Theo định lý Viete ta có $x_1+x_2=2\left(m^2+1\right), x_1 x_2=-2 m$

Ta có $P=x_1+x_2-4 x_1 x_2=$ $2\left(m^2+1\right)-4(-2 m)=2\left(m^2+1+4 m\right)=2(m+2)^2-6 \geq-6$, đẳng thức xảy ra khi $m=-2$.

Bài 8. Trong mặt phẳng tọa độ $O x y$, cho parabol $(P): y=x^2$ và đường thẳng $(d): y=2 x-m-2$. Tìm tất cả các giá trị của tham số $m$ để $(d)$ cắt $(P)$ tại hai điểm phân biệt lần lượt có hoành độ $x_1, x_2$ thỏa mãn $x_1^2+1=2 x_2$.

Lời giải bài 8 .

  • Phương trình hoành độ giao điểm
    $$
    x^2=2 x-m-2 \Leftrightarrow x^2-2 x+m+2=0
    $$
  • Điều kiện $\Delta^{\prime}=1-(m+2)>0 \Leftrightarrow m<-1$.
  • Theo định lý Viete ta có $x_1+x_2=2, x_1 x_2=m+2$.

Ta có $x_1^2=2 x_1-m-2$, suy ra $x_1^2+1=2 x_2 \Leftrightarrow 2 x_1-m-2+1=2 x^2 \Leftrightarrow 2\left(x_1-x_2\right)=m+1$ Kết hợp với Viete ta có $x_1=\frac{m+5}{4}, x_2=\frac{3-m}{4}$
Khi đó $x_1 x_2=m+2 \Leftrightarrow \frac{m+5}{4} \frac{3-m}{4}=m+2 \Leftrightarrow m=-1(l), m=-17(n)$.

  • Vậy $m=-17$.

Bài 9. Cho $(P): y=x^2$ và đường thẳng $(d): y=(m+2) x-2 m$.
a) Tìm $m$ để $d$ cắt $(P)$ tại hai điểm phân biệt $A\left(x_1 ; y_1\right), B\left(x_2 ; y_2\right)$.
b) Tìm $m$ để $x_1+2 y_2=7$.

Lời giải bài 9 .
a) Phương trình hoành độ giao điểm

$\quad x^2-(m+2) x+2 m=0 $
$\Delta=(m+2)^2-8 m=(m-2)^2>0 \Leftrightarrow m \neq 2 .$

b) Khi đó phương trình có nghiệm $x=2, x=m$.
3

  • TH1: $x_1=2, x_2=m$ suy ra $y_1=4, y_2=m^2$. Ta có $2+2 m^2=7$ giải ra được $m=\sqrt{2,5}, m=$ $-\sqrt{2,5}$.
  • TH2: $x_1=m, x_2=2$, suy ra $y_1=m^2, y_2=4$. Ta có $m+2.4=7 \Leftrightarrow m=-1$.
  • Vậy có 3 giá trị $m$ thỏa đề bài $m=\sqrt{2,5}, m=-\sqrt{2,5}, m=-1$.

Bài 10. Trong mặt phẳng tọa độ $O x y$, cho parabol $(P)$ có phương trình $y=x^2$ và đường thẳng $(d)$ có phương trình $y=2 m x-m^2-m-2$ (với $m$ là tham số).
a) Tìm tọa độ điểm $M$ thuộc $(P)$ biết điểm $M$ có hoành độ bằng -3 .
b) Tìm điều kiện của $m$ để đường thẳng $(d)$ cắt parabol $(P)$ tại hai điểm phân biệt. Gọi $A\left(x_1 ; y_1\right), B\left(x_2 ; y_2\right)$ là hai giao điểm của đường thẳng $(d)$ và parabol $(P)$, xác định $m$ để $x_1 y_2+x_2 y_1=2 m^3+6$.

Lời giải bài 10.

b) Tìm điều kiện của $m$ để đường thẳng $(d)$ cắt parabol $(P)$ tại hai đie biệt. Gọi $A\left(x_1 ; y_1\right), B\left(x_2 ; y_2\right)$ là hai giao điểm của đường thẳng $(d)$ và $(P)$, xác định $m$ để $x_1 y_2+x_2 y_1=2 m^3+6$. Ta có phương trình hoành độ giao điểm của $(d)$ và $(P)$ là

$ x^2=2 m x-m^2-m-2 \Leftrightarrow x^2-2 m x+m^2+m+2=0(1) $
$ \Delta^{\prime}=(-m)^2-\left(m^2+m+2\right)=-m-2$

$(d)$ cắt parabol $(P)$ tại 2 điểm phân biệt khi và chỉ khi phương trình (1) có hai nghiệm phân biệt $\Leftrightarrow \Delta^{\prime}>$ $0 \Leftrightarrow-m-2>0 \Leftrightarrow m<-2(*)$

$ \text { Ta có } x_1+x_2=2 m, x_1 x_2=m^2+m+2 $
$x_1 y_2+x_2 y_1=x_1 \cdot x_2^2+x_2 \cdot x_1^2=x_1 \cdot x_2\left(x_1+x_2\right)=2$ $m\left(m^2+m+2\right) $
$=2 m^3+2 m^2+4 m $
$2 m^3+2 m^2+4 m=2 m^3+6 \Leftrightarrow 2 m^2+4 m-6=0 $

$\Leftrightarrow\left[\begin{array}{l}
m=1 \\\
m=-3
\end{array}\right.$

Đối chiếu (*) vậy $m=-3$.

Đề thi thử vào 10 chuyên toán năm 2023 – Star Education

Thời gian làm bài 150 phút

Đề bài.

Bài 1. (2,5 diểm)
(a) Giải phương trình $3 x^3+x+3+(8 x-3) \sqrt{2 x^2+1}=0$.
(b) Cho phương trinh $(\sqrt{x}+1)\left(x^2-3(m+1) x+2 m^2+5 m+2\right)=0(m$ là tham số). Tìm tất cả các giá trị của m để phương trình có hai nghiệm phân biệt thỏa mãn nghiệm này là bình phương nghiệm kia.
(c) n là số tự nhiên lớn hơn hoạc bằng 4, cho $n$ số thực $a_1 \leq a_2 \leq \cdots \leq a_n$ thỏa mãn $a_1+a_2+\cdots a_n=0$ và $\left|a_1\right|+\left|a_2\right|+\cdots\left|a_n\right|=A$. Chứng minh rằng
$$
a_n-a_1 \geq \frac{2 A}{n}
$$

Bài 2. (1,5 điểm) Xét các số $a, b, c$ khác 0 và đôi một phân biệt sao cho các phương trình sau đây có một nghiệm chung:
$$
a x^3+b x+c=0(1), b x^3+c x+a=0(2), c x^3+a x+b=0(3) .
$$
(a) Chứng minh $a+b+c=0$.
(b) Chứng minh rằng một trong các phương trình này có ba nghiệm (không nhất thiết phân biệt).

Bài 3. $(1,5$ điểm)
(a) Tìm số tự nhiên có hai chũ số sao cho nó bằng tổng bình phương các chũ số của nó.
(b) Tìm tất cả các số nguyên tố p, sao cho p có thể biểu diễn được dưới dạng $\sqrt{\frac{a^2-4}{b^2-1}}$, trong đó a,b là các số nguyên dương.

Bài 4. ( 3,5 điểm) Cho đường tròn $(O ; R)$ và dây cung $B C=R \sqrt{3}$ cố định, $A$ thay đổi trên cung lớn $B C$ sao cho tam giác $A B C$ nhọn. Các đường cao $B D, C E$ cắt nhau tại $H$. Phân giác trong góc $A$ cắt $D E$ và $B C$ lần lượt tại $K, L$.
(a) Tính $\angle B A C$ và $\angle O H C$.
(b) Chứng minh $\frac{A K}{A L}$ không đổi. Tìm vị trí của A để KL lớn nhât, tính giá trị đó theo $R$.
(c) Chứng minh đường thẳng d qua L vuông góc $O A$ tiếp xúc với một đường tròn cố định.
(d) Đường thẳng qua K vuông góc DE và đường thẳng qua L vuông góc $B C$ cắt nhau tại P. Chứng minh AP luôn đi qua một điểm cố định.

Bài 5. (1 điểm) Có 10 viên bi vàng và 10 viên bi xanh được xếp thành một hàng. Chúng minh rằng tồn tại 10 viên bi liên tiếp sao cho số viên bi vàng và xanh bằng nhau.

LỜI GIẢI

Một số bài đường tròn và tiếp tuyến

Bài 1. Cho đường tròn tâm $O$ đường kính $A B$. $C$ là một điểm thuộc đường tròn. $d_1$ và $d_2$ lần lượt là tiếp tuyến tại $A$ và $B$ của $(O)$. Tiếp tuyến tại $C$ cắt $d_1, d_2$ lần lượt tại $D$ và $E$. $B C$ cắt $d_1$ tại $F$.
a) Chứng minh $d_1 | d_2$ và $D$ là trung điểm của $A F$.
b) Vẽ đường cao $C H$. Chứng minh rằng $A E, B D$ và $C H$ dồng quy tại trung điểm của $C H$.
c) Chứng minh $O F \perp A E$.

Lời giải.

a) $d_1$ là tiếp tuyến tại $A$ nên $O A \perp d_1, d_2$ là tiếp tuyến tại $B$ nên $d_2 \perp O B$, mà $O, A, B$ thẳng hàng, suy ra $d_1 / / d_2$.
Ta có $\angle A C B=90^{\circ}$, suy ra $\angle D C F+$ $\angle D C A=\angle D F C+\angle D A C=90^{\circ}$. (1)
Hơn nữa $D A=D C$ (t/c tiếp tuyến), tam giác $D A C$ cân tại $D$, suy ra $\angle D C A=$ $\angle D A C$. (2)
Từ (1) và (2) ta có $\angle D C F=\angle D F C$, tam giác $D C F$ cân tại $D$.
Vậy $D F=D C=D A$, hay $D$ là trung điểm của $A F$.
b) Gọi $I$ là giao điểm của $B D$ và $A E$. Ta có $A D / / B E$ nên $\frac{B I}{I D}=\frac{E B}{A D}(3)$.
Mặt khác do $A D=D C$ và $E B=E C$, suy ra $\frac{E B}{A D}=\frac{E C}{D C}$ (4).

Từ (3) và (4) ta có $\frac{B I}{I D}=\frac{E C}{D C}$, suy ra $I C / / A D$ (Thalet đảo).

Mà $A D \perp A B$ nên $C I \perp A B$, vậy $C, I, H$ thẳng hàng.

Do đó $A E, B E, C H$ đồng quy tại $I$.
Ta có $\frac{C I}{A D}=\frac{E I}{E A}, \frac{I H}{A D}=\frac{B I}{B D}$ và $\frac{E I}{E A}=$ $\frac{B I}{B D}$, nên $\frac{C I}{A D}=\frac{I H}{A D}$, suy ra $I C=I H$ hay
$I$ là trung điểm của $C H$.
c) Ta có $E B \cdot A D=E C \cdot C D=O C^2=R^2$, mà $A F=2 A D$ nên $E B \cdot A F=2 R^2$.

Suy ra $E B \cdot A F=A O \cdot A B$, suy ra $\frac{E B}{A B}=\frac{O A}{A F}$, do đó $\tan E A B=\tan A F O$, suy ra $\angle E A B=$ $\angle A F O$.
Mà $\angle E A B+\angle E A F=90^{\circ}$ nên $\angle E A B+$ $\angle A F O=90^{\circ}$. Do đó $O F \perp A E$.

Bài 2. Cho đường tròn tâm $O$ bán kính $R$. $A$ là một điểm nằm ngoài đường tròn, từ $A$ dựng các tiếp tuyến $A B, A C$ dến $(O)$ với $B, C$ là các tiếp điểm. Một cát tuyết qua $A$ cắt $(O)$ tại $D$ và $E$ trong đó $D$ nằm giữa $A$ và $E$.Gọi $H$ là giao điểm của $O A$ và $B C$.
a) Chứng minh $O H \cdot O A=R^2$.
b) Gọi $M$ là trung điểm của $D E$. Chứng minh 4 điểm $O, M, B, C$ cùng thuộc đường tròn.
c) Tiếp tuyến tại $D$ và $E$ của $(O)$ cắt nhau tại điểm $P$. Chứng minh $P, B, C$ thẳng hàng.

Lời giải.

a) Ta có $A B, A C$ là tiếp tuyến nên $A B=A C$, và $O B=O C=R$, suy ra $O A$ là trung trực của $B C$, suy ra $O A \perp B C$ tại $H$.
Tam giác $O A B$ có $\angle O B A=90^{\circ}$ (t/c tiệp tuyến) và $B H \perp O A$ nên $O H \cdot O A=O B^2=$ $R^2$.
b) $M$ là trung điểm $D E$, suy ra $O M \perp D E$.
Ta có $\angle O B A=\angle O M A=\angle O C A=90^{\circ}$, suy ra 5 diểm $O, M, B, A, C$ cùng thuộc đường tròn đường kính $O A$.
c) Ta chứng minh được $O P \perp D E$, suy ra $O, M, P$ thẳng hàng và $O M . O P=O D^2=$ $R^2$.
Suy ra $O M \cdot O P=O H \cdot O A$, suy ra $\frac{O M}{O H}=$ $\frac{O P}{O A}$.
Xét tam giác $O M A$ và tam giác $O H P$ có:
$\angle A O P$ chung $\frac{O M}{O H}=\frac{O P}{O A}$ $\angle O H P=\angle O M A=90^{\circ}$.
Ta có $B C, P H$ vuông góc với $O A$ tại $H$ nên $P, B, C$ thẳng hàng.

Bài 3. Cho tam giác $A B C$ vuông tại $A(A B<A C)$. Vẽ đường tròn tâm $O$ đường kính $A C$ cắt cạnh $B C$ tại $D$. Gọi $H$ và $K$ lần lượt là trung điểm của hai cạnh $A D$ và $C D$. Tia $O H$ cắt cạnh $A B$ tại $E$. Tia $O K$ cắt đường thẳng $E D$ tại $N$ và cắt đường tròn tâm $O$ tại $I$.
(a) Chứng minh $D E$ là tiếp tuyến của $(O)$.
(b) Chứng minh $O H D K$ là hình chữ nhật.
(c) Chứng minh tia $D I$ là tia phân giác của $\angle N D C$.
(d) Gọi $S$ là giao điểm của $O B$ với $A D$. Từ $S$ vẽ đường thẳng vuông góc với $A O$ và cắt tia $O H$ tại $Q$. Chứng minh 3 điểm $A, Q, N$ thẳng hàng.

Lời giải.

Hình 1

a) $OH$ là trung trực của $AD$, suy ra $EA = ED$. Từ đó $\triangle EDO = \triangle EAO (ccc)$, suy ra $\angle EDO = \angle EAO = 90^\circ$. Do đó $ED$ là tiếp tuyến của $(O)$.

b) Do $K$ là trung điểm $CD$ nên $OK \bot CD$, tứ giác $OHDK$ có $\angle D = \angle H = \angle K = 90^\circ$ nên là hình chữ nhật.

c) Ta có tam giác $ODI$ cân tại $O$ nên $\angle ODI = \angle OID$ (1)
Mà $\angle ODI = \angle ODK + \angle KDI, \angle OID = \angle OND + \angle NDI$ (2)
Và $\angle OND = \angle ODK$ (vì cùng phụ $\angle DON$) (3)
Từ (1), (2), (3) ta có $\angle KDI = \angle NDI$

d) Gọi $L$ là giao điểm $AQ$ và $OS$.
Trong tam giác $ASO$ có $AQ, SQ$ là các đường cao, nên $Q$ là trực tâm, suy ra $AQ \bot OS$ tại $L$. (4)
Ta có $OL \cdot OB = OA^2$
và $OK \cdot ON = OD^2 = OA^2$
Suy ra $\angle OK \cdot ON = OL \cdot OB$
Suy ra $\triangle OLN \backsim \triangle OKB$, suy ra $\angle OLN = \angle OKB = 90^\circ$ (5)
Từ (4), (5) ta có $A, L, N$ thẳng hàng, hay $A, Q, N$ thẳng hàng.

Bài 4. Cho đường tròn $(O ; R)$ và một điểm $S$ nằm ngoài đường tròn $(O)$. Vẽ hai tiếp tuyến $S B, S C$ đến $(O)$ với $B, C$ là hai tiếp điểm. Gọi $H$ là giao điểm của $S O$ với $B C$.
(a) Vẽ đường kính $B A$ của $(O)$. Chứng minh $A C || S O$ và $H B \cdot H C=H O \cdot H S$.
(b) Vẽ đường thẳng $d$ vuông góc vớ $A B$ tại $O$, đường thẳng $d$ cắt đường thẳng $A C$ tại $E$. Chứng minh $S E=R$.
(c) Vẽ $C K$ vuông góc với $A B$ tại $K$. Gọi $I$ là trung điểm của cạnh $C K$. Chứng minh 3 điểm $S, I, A$ thẳng hàng.

Lời giải.

a) Do $AB$ là đường kính của $(O)$ nên $\angle ACB = 90^\circ$. (1)

Ta có $SB = SC$ và $SO$ phân giác $\angle BSC$ nên $SO$ là trung trực của $BC$, do đó $OS \bot BC$ tại $H$.

Từ đó ta có $AC ||OS$ vì cùng vuông góc $BC$.

b) $\triangle AOE = \triangle OBS (gcg)$, suy ra $OE = BS$.

Tứ giác $OESB$ có $OE||BS$ (Cùng vuông góc $AB$), và $OE = BS$ nên $OESB$ là hình bình hành, hơn nữa có $\angle OBS= 90^\circ$ nên là hình chữ nhật, do đó $SE = OB = R$.

c) Ta có $OASE$ là hình bình hành, suy ra $AS$ cắt $OE$ tại trung điểm $T$ của mỗi đoạn.
$CK ||OE$
Gọi $I’$ là giao điểm của $AS$ và $CK$
Ta có $\dfrac{I’K}{OT} = \dfrac{AI’}{AT} = \dfrac{CI’}{ET}$
Mà $OT = ET$ nên $KI’ = CI’$, hay $I’ \equiv I$
Vậy $A, I, S$ thẳng hàng

Bài 5. Cho đường tròn $(O ; R)$ và điểm $M$ ở ngoài đường tròn $(O)$. Kẻ tiếp tuyến $M A, M B$ đến $(O)$ với $A, B$ là hai tiếp điểm. Đường thẳng $A B$ cắt $(O)$ tại $K$.
(a) Kẻ đường kính $A N$ của $(O), B H \perp A N$ tại $H$. Chứng $\operatorname{minh} M B \cdot B N=B H \cdot M O$.
(b) Đường thẳng $M O$ cắt đường tròn $(O)$ tại $C$ và $D(C$ nằm giữa $O$ và $M)$. Chứng minh $O K \cdot M K=C K \cdot D K$.
(c) $E$ đối xứng với $C$ qua $K$. Chứng minh $E$ là trực tâm của tam giác $A B D$.
(d) Chứng minh $\sin \angle M^{\circ} A B=\frac{C K}{A K}+\frac{C K}{A M}$

Lời giải.

a) Chứng minh tam giác $OMB$ và $NBH$ đồng dạng.
b) $OK \cdot MK = AK^2 = KC \cdot KD$
c) $ACBE$ là hình thoi, suy ra $BE||AC$, mà $AC \bot AD$ suy ra $BE \bot AD$
$DE \bot AB$
Do đó $E$ là trực tâm tam giác $ABD$.

d) $\angle CAK = \angle CAM$ (chứng minh ở bài trên)
Do đó $\dfrac{CK}{CM} = \dfrac{AK}{AM}$, suy ra $\dfrac{CK}{AK} = \dfrac{CM}{AM}$
Từ đó $VP = \dfrac{CK}{AK} + \dfrac{CK}{AM} = \dfrac{CM}{AM} + \dfrac{CK}{AM} = \dfrac{KM}{AM} = \sin MAB$

Bài 6. Cho hình vuông $A B C D$ cạnh $a, E$ là cung thuộc cung nhỏ $B D$ của đường tròn tâm tâm $A$ bán kính $a$. Tiếp tuyến tại $E$ cắt $C D$ tại $F$ và $B C$ tại $G$.
(a) Chứng minh chu vi tam giác $C F G$ bằng $2 a$.
(b) $A F, A G$ cắt $B D$ tại $I$ và $H$. Chứng minh $H E=$ $H B, I E=I D$

và $H I^2=D I^2+B H^2$
(c) Chứng minh $F H, G I$ và $A E$ đồng quy.

Lời giải.

a) $CD, CB, FG$ là tiếp tuyến của $(A;a)$
Suy ra $FE = FD, GE = GB$
$P_{CFG} = CF + FG + CG = CF + EF +EG+CG = CF+DF +GB+CG = CD+ CB = 2a$

b) $AF$ là trung trực $DE$, và $AG$ là trung trực $BE$
Suy ra $IE = ID, HB = HE$
$\triangle IEF = \triangle IDF \Rightarrow \angle IEF =\angle IDF = 45^\circ$
Tương tự cũng có $\angle HEG = 45^\circ$
Suy ra $\angle IEH = 90^\circ$
Áp dụng pitago cho tam giác $EIH$ ta có $IH^2 = IE^2 + HE^2 = ID^2 + HB^2$

c) Ta có $AF$ là phân giác $\angle DAE$, $AG$ là phân giác của $\angle BAE$
Suy ra $\angle FAG = \dfrac{1}{2} \angle BAD = 45^\circ$.
$\triangle AIH \backsim \triangle DIF (gg)$, suy ra $IA \cdot IF = ID \cdot IH$
Suy ra $\triangle IFH \backsim \triangle IDA \Rightarrow \angle IFH = \angle IDA = 45^\circ$
Suy $\angle AHF = 90^\circ$ hay $FH \bot AG$.
Chứng minh tương tự $GI \bot AF$.
Tam giác $FG$ có $AE, FH, GI$ là các đường cao nên đồng quy.

Bài 7. (Cuối khóa 1 – Star Education 2018) Cho đường tròn $(O ; R)$ và điểm $A$ nằm ngoài đường tròn. Từ $A$ vẽ các tiếp tuyến $A B, A C$ dến $(O)$ ( $B, C$ là các tiếp điểm). $O A$ cắt $B C$ tại $H$.
a) Chứng minh $O H \cdot O A=R^2$ và 4 điểm $O, A, B, C$ cùng thuộc một đường tròn.
b) Đường tròn tâm $I$ đường kính $A B$ cắt $(O)$ tại điểm $D$ khác $B$. Chứng minh $I D$ là tiếp tuyến của $(O)$.
c) Tiếp tuyến tại $H$ và tại $A$ của $(I)$ cắt nhau tại $P$. Chứng minh $B, D, P$ thẳng hàng.

d) Tiếp tuyến tại $H$ của $(I)$ cắt $O B$ tại $M$; gọi $N$ là trung điểm $P M$, đường thẳng qua $P$ song song $B N$ cắt $A B$ tại $K$. Chứng minh $H K, A M$ và $B D$ đồng quy.

Lời giải.

a)

Xét $\triangle A B O$ vuông tại $B$ có:

$B H$ là đường cao $\Rightarrow O H \cdot O A=O B^2=R^2$ (Hệ thức lượng)

Ta có: $\triangle A B O$ vuông tại $B \Rightarrow A, B, O$ thuộc đường tròn đường kính $A O$. (1)

Lại có $\triangle A C O$ vuông tại $C \Rightarrow A, C, O$ thuộc đường tròn đường kính $A O$. (2)

Từ (1) và (2) suy ra $A, B, O, C$ thuộc đường tròn đường kính $A O$.

b)

Ta có: $\triangle A B D$ nội tiếp đường tròn đường kính $A B \Rightarrow \triangle A B D$ vuông tại $D$

Mà $I$ là trung điểm cạnh huyền $A B \Rightarrow I B=I D$
Ta có: $I B=I D, O B=O D$ nên $I O$ là trung trực của $B D$ $\Rightarrow \angle I B O=\angle I D O=90^{\circ}$ nên $I D$ là tiếp tuyến của $(O)$.

c) Tiếp tuyến tại $H$ và tại $A$ của $(I)$ cắt nhau tại $P$. Chứng minh $B, D, P$ thẳng hàng.

Gọi $E=I P \cap A H$ và $F=I O \cap B D$.
Sử dụng tính chất hai tiếp tuyến cắt nhau và hệ thức lượng, ta chứng minh được
$$
I E \cdot I P=I A^2=I D^2=I F \cdot I O \Rightarrow \frac{I F}{I P}=\frac{I E}{I O}
$$

Từ đó, chứng minh được $\triangle I F P \backsim \triangle I E O$ (c.g.c)
$$
\Rightarrow \angle I E O=\angle I F P=90^{\circ} \text {. }
$$

Ta có: $B D$ đi qua $F$ và vuông góc $I O, F P$ đi qua $F$ và vuông góc $I O$ nên hai đường thẳng này trùng nhau. $\Rightarrow B, D, P$ thẳng hàng.

d)

Chứng minh $I H$ là đường trung bình của $\triangle A B C \Rightarrow I H || A C$. Mà $I H \perp P M$ và $A C \perp O C$.

Suy ra: $H M || O C$. Lại có $H$ là trung điểm $B C$ nên $M$ là trung điểm $O B$.

Gọi $Q$ là giao điểm của $P K$ và $B O$.
Ta có: $B N || P Q$ và $N$ là trung điểm của $P M$ nên suy ra $B$ là trung điểm của $Q M$.

Gọi $J=B P \cap A M$.
Ta có :
$ B Q ||A P \Rightarrow \frac{B K}{K A}=\frac{B Q}{P A}=\frac{B M}{P A} . $
$B M || A P \Rightarrow \frac{B M}{P A}=\frac{B J}{J P}$
Suy ra: $\frac{B K}{K A}=\frac{B J}{J P}$ nên $K J || A P$. Chứng minh tương tự $J H ||A P$. Từ đó ta có $K, J, H$ thẳng hàng.

Vậy $H K, B P, A M$ dồng quy tại $J$.

Bài tập luyện tập.

Bài 6. Cho tam giác $A B C$ nhọn. Các đường cao $A D, B E$ và $C F$ cắt nhau tại $H$. Gọi $M, N$ lần lượt là trung điểm của $B C$ và $A H$.
(a) Chứng minh $N E, N F$ là tiếp tuyến của đường tròn ngoại tiếp tam giác $B C E$.
(b) Chứng minh 5 điểm $D, E, F, M, N$ cùng thuộc một đường tròn.
(c) Gọi $G$ là giao điểm của $A D$ và $E F$. Chứng minh $N G \cdot N D=N A^2$.

Bài 7. Cho nửa đường tròn tâm $O$ đường kính $A B=2 R$. Trên tiếp tuyến tại $A$ của $(O)$ lấy điểm $C$ sao cho $A C=A B$. Từ $C$ vẽ tiếp tuyến $C D$ dến $(O)$ cắt tiếp tuyến tại $B$ ở điểm E.
(a) Tính $B E$.
(b) Đường cao $D F$ của tam giác $A B D$ cắt $B C$ tại $G$. Chứng minh rằng $A, G, E$ thẳng hàng.
(c) Gọi $H$ là giao điểm của $O C$ và $A D$. Tính $\angle D H B$.
(d) Gọi $I$ là giao điểm của $B C$ và $(O)$. Tứ giác $I D B H$ là hình gì? Tại sao?

Bài 8. Cho tam giác $A B C$ nhọn nội tiếp đường tròn $(O) . M$ là trung điểm $B C$. Từ $A$ dựng các tiếp tuyến đến đường tròn $(O ; O M)$ cắt $B C$ tại $D$ và $E$ sao cho $D$ và $C$ khác phía đối với $M ; E, B$ khác phía đối với $M$. Chứng minh rằng các tam giác $A D C$ và $A B E$ cân.

Bài 9. Cho tam giác $A B C$ vuông tại $A, A B=a, B C=2 a$. Đường cao $A H$. Từ $B, C$ vẽ các tiếp tuyến $B D, C E$ dến đường tròn tâm $A$ bán kính $A H$.
(a) Tính $A H$ và số đo $\angle A B C$.
(b) Chứng minh $D, A, E$ thẳng hàng.
(c) Chứng minh $E D$ là tiếp tuyến của đường tròn đường kính $B C$.
(d) Chứng minh $D C, B E$ và $A H$ dồng quy.

Bài 10. Cho hình vuông $A B C D$ cạnh $2 a$, tâm $O$. Đường tròn tâm $O$ bán kính $a$ tiếp xúc với $A B$ và $B C$ tại $E$ và $F$. Gọi $P$ là một điểm trên cung nhỏ $E F$. Tiếp tuyến tại $P$ cắt $A B, B C$ tại $M$ và $N$. Đặt $M B=c, B N=y$.
(a) Chứng minh rằng $x+y+\sqrt{x^2+y^2}=2 a$.
(b) Chứng minh rằng $A M \cdot C N=2 a^2$.
(c) Gọi $K$ là trung điểm của $A D$. Chứng minh rằng $M K |$ $D N$.